You are on page 1of 458

CALIBRATE REVIEW CENTER

Final Coaching
April 15, 2022
Live from Santa Barbara, California

Marleth Picones, RMT, MLS (ASCPi)CM

1
EXAMINATION CONTENT AREAS
CONTENT AREA DESCRIPTION EXAM PERCENTAGE
Blood Banking Blood Products, Blood Group Systems, Blood Group
Immunology, Physiology and Pathophysiology, Serology and 17-22%
Molecular Testing, Transfusion Practice

Urinalysis and Physical and Chemical Testing, Microscopic Analysis, Physiology, 5-10%
Other Body Fluids Disease States

Chemistry Carbohydrates, Lipids, Heme Derivatives, Enzymes, Proteins &


Other Nitrogen‐Containing Compounds, Acid‐Base
17-22%
Determinations (Including Blood Gases), Electrolytes,
Endocrinology, Vitamins and Nutrition, Therapeutic Drug
Monitoring, Toxicology

2
CONTENT AREA DESCRIPTION EXAM PERCENTAGE
Hematology Physiology, Disease States, Hematology Laboratory Testing,
Hemostasis 17-22%
Immunology Principles of Immunology, Diseases of the Immune System, 5-10%
Transplantation, Infectious Disease Serology, Serologic and
Molecular Procedures, Test Results

Microbiology Preanalytic Procedures; Analytic Procedures for Bacteriology;


Analytic Procedures for Mycology, Mycobacteriology, 17-22%
Parasitology, and Virology; Post‐Analytic Procedures

Laboratory Quality Assessment/Troubleshooting, Safety, Laboratory 5-10%


Operations Mathematics, Manual/Automated Methodology and
Instrumentation, Basic Management Principles, Education
Principles

3
TOPICS
I. BLOOD BANK 41
II. Urinalysis and Other Body Fluids 20
III. CHEMISTRY 47
IV. HEMATOLOGY 35
V. COAGULATION 15
VI. IMMUNOLOGY 16
VII. MICROBIOLOGY 35
VIII. MYCOLOGY 10
IX. PARASITOLOGY 15
X. General / Laboratory Operations 15
XI. ROAD TO USA TIPS
**I-X are mostly recalls

4
I. BLOOD BANK

5
1. Mother is group A positive, Father is group
O negative, the baby experienced HDN,
what caused the hemolytic disease of the
newborn?
a. Anti-D
b. Anti-d
c. Anti-A
d. Anti-c
Answer: Anti-c
Mom does not have Anti-D as she is Rh positive.
Anti-A does not cause HDN. Anti-d does not exist.

Therefore it is Anti-c

Blood bank 6
Donor Blood typing results
Anti-A Anti-B A cells B cells D antigen D control
4+ 4+ 0 0 0 0

2. What to do next? Answer:


B. Perform Du testing

A. Do nothing • AABB requires that weak D testing be performed on


all donors that appear to be D negative at IS
B. Perform Du testing • Goal: Prevent anti-D immunization in recipients
C. Report as D positive (donors)

D. Perform Ab screen • Donor centers routinely perform weak D testing

Blood bank 7
3. All donor testing must include:

a. Complete Rh phenotyping
b. Anti-CMV testing
c. Direct antiglobulin test
d. Serological test for syphilis

Answer: D. Serological test for syphilis


Testing for syphilis was the first mandated donor
screening test for infectious disease and is still part of
donor screening (AABB tech manual 2008b, ch8)
Blood bank 8
4. The Western blot is a confirmatory test
for the presence of:

a. CMV Antibody
b. Anti-HIV-1
c. HBsAg
d. Serum protein abnormalities
Answer: B. Anti-HIV-1
Western blot uses purified HIV proteins to confirm
reactivity in samples whose screening test for anti-HIV
is positive.
Blood bank 9
REMEMBER!

SNOW DROP

Southern blot DNA


Northern blot RNA
o o
Western blot Proteins

Blood bank 10
5. When platelets are stored on a rotator set on
an open bench top, the ambient air temperature
must be recorded:

a. Once a day
b. Twice a day
c. Every 4 hours
d. Every hour
Answer: C. Every 4 hours
If storage devices do not have automated temperature recording,
temperature must be manually monitored every 4 hours. (AABB
Tech manual 2008b, p284)

Blood bank 11
6. A unit of Red Blood cells is issued at 9:00am. At 9:10am
the unit is returned to Blood bank. The container has not been
entered, but the unit has not been refrigerated during this time
span. The best course of action for the technologist is to:

a. Culture the unit for bacterial contamination


b. Discard the unit if not used within 24 hours
c. Store the unit at room temperature
d. Record the return and place the unit back into inventory

Answer: D. Record the return and place the unit back into inventory
Studies have shown that refrigerated components retain an acceptable temperature of
<10C for up to 30 minutes after removal from the refrigerator (AABB Tech manual 2008b)

Blood bank 12
7. The optimum storage temperature for Red
Blood Cells, Frozen is:

a. -80C
b. -20C
c. -12C
d. 4C
Answer: A. -80C
Red Blood Cells, Frozen with 40% glycerol are stored at -65C or lower (AABB Standards
2008a, Reference standard 5.1)

Blood bank 13
8. What is the AABB age requirement for blood
donation?

a. 16 years old
b. 17 years old
c. 18 years old
d. 19 years old

Answer: b. 17 years old


Blood donors must be at least 17 years of age or the age stipulated by
applicable state law. (AABB Standards 2005, page 98)

Blood bank 14
9. The optimum storage temperature for
platelets is:

a. -20C
b. -12C
c. 4C
d. 22C
Answer: D. 22C
The required temperature for storage of platelets is 20-24C (AABB Standards
2008a, Reference standard 5.1)

Blood bank 15
10. According to AABB standards, Fresh Frozen
Plasma must be infused within what period of
time following thawing?

a. 24 hours
b. 36 hours
c. 48 hours
d. 72 hours
Answer: A. 24 hours
Per AABB standards, thawed FFP should be stored at 1-6C for no more than
24 hours (AABB Standards 2008a, Reference standard 5.1)

Blood bank 16
11. Once thawed, Fresh Frozen Plasma must
be transfused within:

a. 4 hours
b. 8 hours
c. 12 hours
d. 24 hours

Answer: D. 24 hours
Thawed FFP is stored at 1-6C for up to 24 hours (Marques 2007, p25)

Blood bank 17
12. What is the patient’s blood type?
FRONT TYPE BACK TYPE
Anti –A Anti-B Rh Rh control A1 Cells B cells
a. O positive
MF 4+/0 MF 4+/0 4+ 0 0 0
b. AB positive Note: Patient received 10 units of O pos RBCs
c. A positive
d. O negative

Answer: B. AB positive
Patient is type AB, but received several type O RBC units. Both show up
in the front type.

Blood bank 18
13. Each unit of whole blood will yield approximately
how many units of Cryoprecipitated AHF?
a. 40
b. 80
c. 130
d. 250

Answer: B. 80
Cryoprecipitate contains at least 80 units of Antihemophilic Factor (Harmening
2005, p237)

Blood bank 19
14. According to AABB standards, platelets
must be:
a. Gently agitated if stored at room temperature
b. Separated within 12 hours of Whole Blood collection
c. Suspended in sufficient plasma to maintain a pH of 5.0 or lower
d. Prepared only from whole blood unit that have been stored at 4C
for 6 hours

Answer: A. Gently agitated if stored at room temperature


Per AABB standards, store platelets at 20-24C with continuous agitation.
Platelets must be separated from Whole Blood units and maintained at a
temperature of at least 20C. The pH must be at least 6.2 at the end of storage
time. (AABB standards 2008a, p65))
Blood bank 20
15. An individual has been sensitized to the k
antigen and has produced anti-k. What is her
most probable Kell system genotype?
a. KK
b. Kk
c. Kk
d. K0K0

Answer: A. KK
This individual cannot have the k antigen on their cells. K0K0 is rare and no
Kell system antigens are detected on the red blood cells. (Harmening 2005, p176)

Blood bank 21
16. Patient receiving blood from mother
requires what type of blood? (no choices)
Answer:
Irradiated Blood
Relatives might probably have similar HLA which will recognized by the baby's immune system as
same, hence won't be attacked.
May cause Graft vs Host disease. Hence, Leukocyte need to be irradiated

Blood bank 22
17. What disease or infection is this associated with?
What would cause this blood picture?

Answers: Mycoplasma pneumoniae and Cold reacting antibodies


Blood bank 23
Autoantibodies • Cold, warm or both
• React with all cells, including patient’s own
• Uniform strength

COLD WARM

IgM IgG
Usually react best at RT and colder, sometimes React best at AHG
strong enough to carry over into AHG phase
PEG and enzymes will enhance
Will interfere with RT testing (ISXM)
LISS doesn’t enhance
M, N, Le, P, I, IH, Lu
Will interfere with all testing that includes
AHG phase
Gel
18. Most blood type are inherited as?
a. Sex-linked dominant
b. Sex-linked recessive
c. Autosomal recessive
d. Autosomal codominant

Answer: D. Autosomal codominant


Blood group genes are autosomal, they are not carried on the sex gene.
Whenever the gene is inherited, the antigen is expressed on the red cells,
which is known as codominant. (Harmening 2005, p110)
Codominant example:
Mom: A1o
Dad: BO
Baby: A1B
Blood bank 25
19. Anti-N is identified in a patient’s serum. If random
crossmatches are performed on 10 donor units, how
many would be expected to be compatible?
a. 0
b. 3
c. 7
d. 10

Answer: B. 3
The N antigen is lacking in 30% of the Caucasian population. (AABB Tech manual
2008b, p415)

Blood bank 26
Blood bank 27
20. A donor is tested with Rh antisera with the following results?
Anti-D Anti-C Anti-E Anti-c Anti-e Rh control
+ + 0 + + 0

What is the probable Rh genotype?


a. R1R1
b. R1r
c. R0r
d. R2r

Answer: B. R1r
The haplotype is DCe/dce. (AABB Tech manual 2008b, p391)

Blood bank 28
REVIEW!
Wiener
❖Believed that a single gene encoded several blood group factors
❖Each gene produces a structure on the red cell called an agglutinogen (antigen)
❖Eight major alleles (agglutinogens): R0, R1 , R2 , R z , r , r ’ , r “ ,ry RD
❖R: D antigen present r  no D
❖r: D antigen absent (d)
1 and ‘  C
❖Subscripts and superscripts
o 0 or nothing: have c and e, no C or E 2 and “  E
o 1 or ‘ : have C and e, no c or E
o 2 or “ : have c and E, no C or e
o Letter (z and y): have C and E, no c or e Example: DcE  R2
r”  dcE

Blood bank
Common Rh types by nomenclature
Table 7-5 Harmening Wiener Fisher-Race Rosenfield
Phenoytpe
Common Genotypes R1 r DCe / dce 1, 2, -3, 4, 5
R1 R1 DCe / DCe 1, 2, -3, -4, 5
rr dce / dce -1, -2, -3, 4, 5
R1 R2 DCe / DcE 1, 2, 3, 4, 5
R2 r DcE / dce 1, -2, 3, 4, 5
R2 R2 DcE / DcE 1, -2, 3, 4, -5

Blood bank
The Big 4 –
“White 120” = R1>r>R2>R0
“Black Or 12” = R0>r>R1>R2

Blood bank 31
21. Which of the following red cell antigens are
found on glycophorin-A?
a. M,N
b. Lea, Leb
c. S, s
d. P, P1, Pk

Answer: A. M,N
The M and N antigens are found on glycophorin A. (Harmening 2005, p167)

Blood bank 32
REVIEW!
MNS system
M and N U
On Glycophorin A (GPA) On GPB
On cord cells
Differ by 2 amino acids
U neg only seen when S-s-
On cord cells
Receptors for complement, viruses and bacteria M, N are destroyed by enzymes

S and s S and s are less easily degraded by enzymes


On Glycophorin B (GPB)
On cord cells
Differ by one amino acid
Receptors for complement, viruses and bacteria

Blood bank 33
22. Anti-Fya is:
a. Usually a cold-reactive agglutinin
b. More reactive when tested with enzyme-treated red blood cells
c. Capable of causing hemolytic transfusion reactions
d. Often an agglutinin

Answer: C. Capable of causing hemolytic transfusion reactions


Anti-Fya is an IgG antibody that reacts best at the AHG phase, does not react
with enzyme-treated red cells, is capable of causing hemolytic disease of the
newborn, and is not known to be an autoagglutinin. (Harmening 2005, pp180-181)

Blood bank 34
23. Resistance to malaria is best associated with
which of the following blood groups:
a. Rh
b. I/i
c. P
d. Duffy

Answer: D. Duffy
Th e duffy glycoprotein on red cells is a receptor for the malarial parasite
Plasmodium vivax. Red cells with the phenotype Fy(a-b-) are resistant to
invasion by P. vivax. (Harmening 2005, p182)

Blood bank 35
24. What is used to neutralize P1 antibody?
(no choices)
Answer:
Hydatid cyst fluid

The discovery of strong anti-P1 in two P1–individuals infected with Echinococcus


granulosus tapeworms led to the identification of P1 and Pk substance in hydatid cyst fluid.

Soluble P1 substances have potential use in the blood bank and are commercially available.
When it is necessary to confirm antibody specificity or to identify underlying antibodies, these
substances can be used to neutralize anti-P1. (Harmening)

Blood bank 36
25. A man suffering from gastrointestinal bleeding has received 20
unites of Red Blood Cells in the last 24 hours and is still oozing post-
operatively. The following results were obtained :

PT: 20 seconds (control: 12 seconds)


aPTT: 43 seconds (control: 31 seconds)
Platelet count: 160 x 103/uL
Hgb: 10 g/dL
Factor VIII: 85%

What blood product should be administered?


a. FFP
b. RBCs
c. Factor VIII concentrate
d. Platelets

Blood bank 37
#25. ANSWER:

A. Fresh Frozen Plasma

Massive transfusion patients (2 or more blood volume)


usually require platelets and FFP but since his platelet count
is adequate, only FFP should be given at this time.

(Harmening 2005, p314)

Blood bank 38
26. ABO-hemolytic disease of the newborn:

a. Usually requires an exchange transfusion


b. Most often occurs in first born children
c. Frequently results in stillbirth
d. Is usually seen only in the newborn of group O mothers

ANSWER: D. Is usually seen only in the newborn of group O mothers


ABO Hemolytic Disease of the Fetus and Newborn (ABO HDFN) is a mild
disease, not usually requiring transfusion. It may occur in any pregnancy in
which there is ABO incompatibility. High titered IgG antibodies are more
frequently seen in group O mothers.

Blood bank 39
27. Which of the following antigens is most likely to be involved in
hemolytic disease of the newborn?

a. Lea
b. P1
c. M
d. Kell

ANSWER: D. Kell
HDFN is caused by maternal IgG antibodies. Outside the Rh system, the
most clinically significant antibody for HDFN is anti-K. IgM antibodies do not
cross placenta. (Harmening 2005, p385)

Blood bank 40
Blood bank 41
• Cold, warm or both
Autoantibodies • React with all cells, including patient’s own
• Uniform strength

COLD WARM

IgM IgG
Usually react best at RT and colder, sometimes React best at AHG
strong enough to carry over into AHG phase
PEG and enzymes will enhance
Will interfere with RT testing (ISXM)
LISS doesn’t enhance
M, N, Le, P, I, IH, Lu
Will interfere with all testing that includes
AHG phase
Gel

Blood bank
28. A blood specimen from a pregnant woman is found to be
group B, Rh-negative and the serum contains Anti-D with a titer of
512. What would be the most appropriate type of blood to have
available for a possible exchange transfusion?

a. O, Rh negative
b. O, Rh positive
c. B, Rh negative
d. B, Rh positive

ANSWER: A. O, Rh negative
Blood selected for exchange transfusion should be ABO-compatible with the
mother and baby, and antigen negative. Prenatal Ab titers above 16 or 32 are
considered significant, fetus should be monitored. (Harmening 2005, pp387-390)
Blood bank 43
29. A fetomaternal hemorrhage of 35mL of fetal Rh-positive
packed RBCs has been detected in an Rh-negative woman. How
many vials of Rh immune globulin should be given?
a. 0
b. 1
c. 2
d. 3

ANSWER: D. 3
One vial of Rh immune globulin protects against a fetomaternal hemorrhage
of 15mL of red cells or 30mL of whole blood.
Volume of fetomaternal hemmorhage (35) / 15 = 2 then add extra vial
2+1= 3
(AABB tech manual 2008b, p632)

Blood bank 44
30. The results of a Kleihauer-Betke stain indicate a fetomaternal
hemmorage of 35mL of whole blood. How many vials of Rh
immune globulin would be required?
a. 0
b. 1
c. 2
d. 3

ANSWER: C. 2

One dose of RhIg will protect the mother from a bleed of 30mL. The bleed
was 35mL, 2 vials of RhIg will be needed.
(AABB tech manual 2008b, pp631-632)

Blood bank 45
31. A Kleihauer-Betke stain of a postpartum blood film revealed
0.3% fetal cells. What is the estimated volume (mL) of the
fetomaternal hemorrhage expressed as whole blood?
a. 5
b. 15
c. 25
d. 35

ANSWER: B. 15

The formula to calculate the percentage assumes the mother’s blood volume
as 5000mL.
0.003 x 5000mL = 15mL
(AABB tech manual 2008b, p632)

Blood bank 46
32. HLA Antigen typing is important in screening for?
a. ABO incompatibility
b. A kidney donor
c. Rh incompatibility
d. A blood donor

ANSWER: B. A kidney donor

HLA Antigen typing is important to consider before organ transplantation.


(Harmening 2005, p227)

Blood bank 47
33. HLA Antibodies are?
a. Naturally occuring
b. Induced by multiple transfusions
c. Directed against granulocyte antigens only
d. Frequently cause hemolytic transfusions reactions

ANSWER: D. Frequently cause hemolytic transfusions reactions

Major Histocompatibility Complex (MHC) consists of both class I and class II


HLA Antigens. Discrimination of self from nonself is the primary function of
the HLA system and involves many immune responses.
(AABB Tech Manual 2008b, p555)

Blood bank 48
34. Saliva from which of the following individuals would neutralize
an auto anti-H in the serum of a group A, Le(a-b+) patient?
a. Group A, Le (a-b-)
b. Group A, Le(a+b-)
c. Group O, Le(a+b-)
d. Group O, Le(a-b+)

ANSWER: D. Group O, Le(a-b+)

Group O have the most H substance in their saliva. The person must also be
a secretor of ABH substances.
(Harmening 2005, p112)

Blood bank 49
35. Which of the following Rh antigens has the highest frequency
in Caucasians?
a. D
b. E
c. c
d. e

ANSWER: D. e

The overall incidence of the e antigen is 98%. The overall incidence of c is


80%, D is 85% and E is 30%.
(Harmening 2005, p136)

Blood bank 50
36. The following test results are noted for a unit of blood labeled
group A, Rh negative?
Cells tested with
Anti-A Anti-B Anti-D
4+ 0 3+

What should be done next?

a. Transfuse as a group A, Rh-negative


b. Transfuse as a group A, Rh-positive
c. Notify the collecting facility
d. Discard the unit

Blood bank 51
#36

ANSWER: C. Notify the collecting facility

A serological test to confirm the ABO on all RBC


units and Rh on units labeled as Rh-negative must
be performed prior to transfusion. Any errors in
labeling must be reported to the collection facility.
(AABB Tech manual 2008b, p451)

Blood bank 52
37. The following results were obtained in pretransfusion testing:
37C IAT
Screening cell I 0 3+
Screening cell II 0 3+
Autocontrol 0 3+

The most probable cause of these results is:

a. rouleaux
b. A warm autoantibody
c. A cold autoantibody
d. Multiple alloantibodies

Blood bank 53
#37

ANSWER: B. A warm autoantibody

Presence of agglutination at AHG phase with both


screening and autocontrol is indicative of warm
autoantibody.
(Harmening 2005, p246)

Blood bank 54
38.

Which clinical condition is consistent with the lab results shown above?

a. Cold hemagglutinin disease


b. Warm autoimmune hemolytic anemia
c. Penicillin-induced hemolytic anemia
d. Delayed hemolytic transfusion reaction

Blood bank 55
#38

ANSWER: b. Warm autoimmune hemolytic anemia

Reaction with anti-IgG in the DAT and with both screening cells
and autocontrol at AHG phase is indicative of warm
autoantibody.

(Harmening 2005, p407)


Blood bank 56
Riddle TIME!

What runs faster, cold or hot?

HOT, because you can catch a cold.

Blood bank 57
39. AHG (Coombs) control cells

a. Can be used as a positive control for anti-C3 reagents


b. Can be used only for the indirect antiglobulin test
c. Are coated only with IgG antibody
d. Must be used to confirm all positive antiglobulin reactions

Answer: C. Are coated only with IgG antibody


AHG control cells are IgG sensitized cells that react with the anti-IgG in the AHG reagent to
demonstrate AHG was added and not neutralized by insufficient washing of the tests prior to its
addition. (AABB Tech manual 2008n, p449)

Blood bank 58
40. A sample gives the following results:
Cells with: Serum with:
Anti-A 3+ A1 cells 2+
Anti-B 4+ B cells 0

Which lectin should be used first to resolve this discrepancy?


a. Ulex europaeus
b. Arachis hypogaea
c. Dolichos biflorus
d. Vicia graminea

Answer: C. Dolichus biflorus


Dolichus biflorus plant seed extract forms complexes with N-acetylgalactosamine. When properly
diluted, it can distinguish between A1 donor cells and all other subgroups of A. (AABB Tech manual
2008b, p365)

Blood bank 59
Lectins
Lectin: carbohydrate binding proteins that are highly specific for certain sugars
Anti-A1 lectin:
Dolichos biflorus
Reacts with A1 subgroup but not A2 subgroup ; Used to differentiate between A1 and A2 subgroups
If there is a negative result with D. biflorus
Proven that the patient is not A1
Proven that the patient has the ability to make anti-A1
Anti-H lectin
Ulex europaeus ; Reacts with H antigen; Bombay will be negative

Bandeiraea simplicifolia: Anti-B lectin


Arachis hypogaea: peanut lectin to differentiate T polyagglutination from Tn polyagglutination
Vicia graminea: Anti-N lectin
41. In the Hardy–Weinberg formula, p2 represents:

A. The heterozygous population of one allele


B. The homozygous population of one allele
C. The recessive allele
D. The dominant allele
ANSWER: B. The homozygous population of one allele

Hardy–Weinberg formula: p2 + 2pq + q2

p2 and q2: homozygous expressions


2pq: heterozygous expression.

This formula is used in population genetics to determine the frequency of different alleles.

Blood bank 61
Hardy-Weinberg formula
p+q=1 OR p2 + 2pq + q2 = 1

Example: DD and Dd (Rh positive) = 84%


dd (Rh negative) = 16%

p2 (DD) + 2pq (Dd) + q2 (dd) = 1

(0.84) + (0.16)2 = 1

p + 0.4 =1

P = 1 - 0.4
P = 0.6

p2 + 2pq + q2 = 1 (0.6) 2 + 2 (0.6) (0.4) + (0.4)2

Therefore, 0.36 (DD) + 0.48 (Dd) + 0.16 (dd) = 1

Blood bank 62
Blood bank 63
42. What is the antibody?

Blood bank 64
43. The serum from #41 is tested against Group O RBCs and
cord cells. Showed no reaction with cord cells. What antibody?

Cold-reacting autoantibodies may be suspected when the


screen cells, panel cells, and the autocontrol are all
positive at the immediate spin phase and reactivity gets
weaker or disappears with incubation at 37°C

Cord blood cells that lack the I antigen are of particular


value for this purpose. A cold panel consisting of group
O adult cells (H and I antigens), group O cord cells (H
and i antigens), group A1 adult cells (I antigen), and an
autocontrol may be tested at 4°C to determine the
specificity of the cold autoantibody

Answer: Anti-I because it did not react with cord cells

Blood bank 65
42. Antibody panel

Blood bank 66
Blood bank 67
Blood bank 68
Blood bank 69
Blood bank 70
Blood bank 71
Zygosity
Some antibodies react stronger when tested with a homozygous panel cell than
with a heterozygous cell.
◦ Showing dosage
◦ Some antibodies can be negative with heterozygous cells

Duffy, Kidd, Rh, MNS


Homozygous: double dose of an antigen
◦ Jk(a+b=) or Jk(a=b+)

Heterozygous: single dose of an antigen


◦ Jk(a+b+)
Homozygous vs. Heterozygous
Homozygous
◦ Expression of the same allele at both of the two chromosome
sites.
◦ Example Fya , Fya

Fya Fyb E e

+ Ø Ø +
Homozygous vs. Heterozygous
Heterozygous
◦ Expression of the different alleles at the same loci on the two
chromosome sites.
◦ Example Fya , Fyb

Fya Fyb E e

+ + + +
Antibody ID Steps-Ruling Out
❖Use both the antibody screen and panel
❖Use only cell lines with negative results
❖Perform cross-outs using homozygous cells
◦ Put a “/” through the reaction of that cell
◦ Exception: Can rule out with homozygous cells in certain situations. Use parentheses around the
reaction on that cell.

❖Continue down negative cell lines with cross-out technique


❖Match pattern if possible

Blood bank 75
Example

Blood bank
Example

Blood bank
Example

Blood bank
Example

Blood bank
Example

Blood bank
Example

Blood bank
Example

Blood bank
Selected Cells
❖Every possible antibody must be ruled in or out
❖Additional cell panel or selected cell panel
❖Select cells based on what antibodies still need to be ruled out
❖Choose homozygous cells if possible

Blood bank
Antigen Typing
Last step in antibody ID process
You haven’t proven that an antibody is present until you’ve proven that the patient has
the ability to make an antibody
◦ Negative for the corresponding antigen

Only perform on patient’s who have not been transfused in the last 3 months
◦ Ensure that you’re testing patient’s cells, not transfused cells
◦ It can be done, but you can’t trust weak or mixed field reactions
◦ There are ways to get rid of transfused cells in order to phenotype patient – reference lab

Controls
◦ Positive control: heterozygous
◦ Anti-Jka: pick a Jk(a+b+) cell

Blood bank
What antibody(ies) are present?
A. anti-c
B. Anti-M
C. Anti-Fya
D. Anti-Jkb
Blood bank 85
TIPS
STUDY THE ANTIBODY PANEL and FREQUENCY OF BLOOD GROUPS

Blood bank 86
Blood bank 87
Blood bank 88
Blood bank 89
II. URINALYSIS AND
OTHER BODY FLUIDS

90
1. Identify this crystal:

a. Ammonium biurate crystals


b. Bilirubin crystals
c. Tyrosine cystals
d. Leucine cystals

Urinalysis and Other Body fluids 91


#1 ANSWER: B. BILIRUBIN CRYSTAL
a. Ammonium biurate crystals : alkaline, yellow-brown; spheres or spheres with spicules (Thorny apples); Normal
b. Bilirubin crystals
c. Tyrosine cystals: Colorless to brown; Fine needles; Abnormal: Liver disease, aminoaciduria
d. Leucine cystals: Dark Yellow/brown; Spheres with radial striations and concentric circles; Abnormal: Liver
disease and aminoaciduria
Description:
❖ Bright yellow to yellow-brown
❖ Fine amorphous needles

Clinical Significance:
Liver disease

(CAP 2018)

Urinalysis and Other Body fluids 92


Bilirubin Crystals

Urinalysis and Other Body fluids 93


2. Identify this crystal

Urinalysis and Other Body fluids 94


#2 ANSWER: Cystine

Description:
Colorless
Six-sided plates with corner notches
Refractile

Significance:
Cystinosis or Cystinuria
Rarely seen, these crystals are found in acid urine

Urinalysis and Other Body fluids 95


96
3. False urobilinogen testing may occur if the
urine specimen is:
a. Exposed to light
b. Adjusted to a neutral pH
c. Cooled to room temperature
d. Collected in a nonsterile container

ANSWER: a. exposed to light

Urobilinogen is degraded by light. (Strasinger 2008, p36)

Urinalysis and Other Body fluids 97


4. A urine specimen comes to the laboratory 7 hours
after it is obtained. It is acceptable for culture only if
the specimen has been stored:
a. At room temperature
b. At 4-7C
c. Frozen
d. With a preservative additive

ANSWER: b. at 4-7C

Refrigeration prevents bacterial growth for 24 hours. (Strasinger 2014, p32)

Urinalysis and Other Body fluids 98


5. Urine that develops a port wine color after
standing may contain:
a. melanin
b. porphyrins
c. bilirubin
d. urobilinogen

ANSWER: b. porphyrins

Colors associated with urine are due to pigments.


Melanin: Black
Bilirubin: amber to brown (Strasinger 2014, pp60-61)

Urinalysis and Other Body fluids 99


6. In which of the following metabolic disease will
urine turn dark brown to black upon standing:
a. phenylketonuria
b. alkaptonuria
c. Maple syrup disease
d. Aminoaciduria

ANSWER: b. alkaptonuria

Phenylketonuria: mousy odor


Maple syrup: maple syrup odor
Aminoaciduria: include phenylketonuria (PKU), tyrosyluria, alkaptonuria, melanuria, maple syrup
urine disease, organic acidemias, indicanuria,
cystinuria, and cystinosis.
(Strasinger 2014, p61)

Urinalysis and Other Body fluids 100


7. Given the following urinalysis results, select the most appropriate
course of action:

pH = 5.0
Protein = Neg
Glucose = 1,000 mg/dL
Blood = Neg
Bilirubin = Neg
Ketone = Moderate
SSA protein = 1+

A. Report the SSA protein result instead of the dry reagent strip result
B. Call for a list of medications administered to the patient
C. Perform a quantitative urinary albumin
D. Perform a test for microalbuminuria

Urinalysis and Other Body fluids 101


#7. ANSWER
B. Call for a list of medications administered to the patient

The sulfosalicylic acid (SSA) test is a cold precipitation test that reacts equally with all forms
of protein.

The combination of glucose- and ketone- positive urine points to a patient with insulin-
dependent diabetes. A false positive SSA test is likely if tolbutamide (Orinase) has been
administered.

Harr 2013 and Strasinger 2014

Urinalysis and Other Body fluids 102


8. Urine production of less than 400 mL/day is:

A. Polyuria
B. Oliguria
C. Anuria
D. Associated with Diabetes mellitus

ANSWER: B. Oliguria
a decrease in urine output (which is less than 1 mL/kg/hr in infants, less than 0.5 mL/kg/hr in
children, and less than 400 mL/day in adults), is commonly seen when the body enters a state
of dehydration as a result of excessive water loss from vomiting, diarrhea, perspiration, or
severe burns.

Urinalysis and Other Body fluids 103


Urinalysis and Other Body fluids 104
9. What stain will help confirm this cast?

a. Sudan black B
b. Toluidine Blue
c. Oil red O
d. Prussian Blue Stain

Picture from lab CE per CHS intern 2020

Urinalysis and Other Body fluids 105


The picture shown was a FATTY cast showing refractile circles. Can be verified by staining with Oil red O and
Sudan III (triglycerides or neutral fats) or examining under polarized light for the presence of maltese-cross
formation (cholesterol). Seen in lipiduria, diabetes mellitus and crush injuries. (CAP 2018 and Strasinger 2014)

ANSWER: C. Oil red O


Stain triglycerides and neutral fats orange-red. Do not stain cholesterol.

Sudan Black B: Stain phospholipids in peripheral smear. Myeloblasts: strong + ; Monoblasts faint +
Toluidine Blue: Enhances nuclear stain; Differentiates WBCs and renal tubular epithelial (RTE) cells
Prussian Blue Stain: Stains structures containing iron; Identifies yellow-brown granules of hemo-
siderin in cells and casts

Urinalysis and Other Body fluids 106


10. Using compensated polarized light, what crystal will positive (blue)
birefringence?

A. Calcium oxalate
B. Calcium pyrophosphate dihydrate
C. Uric acid
D. Cholesterol

ANSWER: B. Calcium pyrophosphate dihydrate


MSU crystal with grain running parallel to the long axis. The slow ray passes with the grain, producing
negative (yellow) birefringence. CPPD crystal with grain running perpendicular to the long axis. The slow
ray passes against the grain and is retarded, producing positive (blue) birefringence.

Monosodium Urates (MSU): parallel yellow (negative birefringence), perpendicular blue.


Calcium pyrophosphate dihydrate (CPPD): parallel blue (positive) birefringence, perpendicular yellow

Urinalysis and Other Body fluids 107


Urinalysis and Other Body fluids 108
11. Which of the following stains is used to determine sperm viability?

A. Eosin Y
B. Hematoxylin
C. Papanicolau
D. Methylene Blue

ANSWER: A. EOSIN Y

Eosin Y is excluded by living sperm and is used to determine the percentage of living
cells. Papanicolau, Giemsa’s and hematoxylin stains are used to evaluate sperm
morphology. The viability test should be performed whenever the results of motility test
are subnormal.

Urinalysis and Other Body fluids 109


12. A urine specimen is tested and has the following results”

Reagent Strip Microscopic findings


Glucose: 3+ >100 WBCs/hpf
Protein: 1+ many yeast cells

This is indicative of:


A. Diabetes mellitus
B. Contamination
C. Pyelonephritis
D. Diabetes insipidus

ANSWER: A. Diabetes mellitus

Yeast cells are commonly seen in urine specimens from persons with diabetes
mellitus, because the high glucose content provides an excellent growth media for
yeast.

Urinalysis and Other Body fluids 110


13. These 3 tubes of cerebrospinal fluid are delivered to the laboratory
for analysis. The tube labeled #1 was the first tube collected. Which one
should be used by hematology department for cell count and
differential?

A. Tube #1
B. Tube #2
C. Tube #3
D. Reject the samples

ANSWER: C. Tube #3
Tube#1 Chemistry and/or Serology
Tube#2 Microbiology
Tube#3 Hematology
(Clinical Laboratory Science Review: a bottom line approach by Patty et. Al)

Urinalysis and Other Body fluids 111


14. Based on the picture from #13 what do you think is the cause of
subsequent clearing of blood in each tube?

A. Traumatic tap
B. Hemorrhage
C. Both
D. None of the above

ANSWER: A. Traumatic tap

Differentiation Between Hemorrhage vs. Traumatic Tap

Hemorrhage Traumatic Tap

Appearance All tubes equally red (bloody) Subsequent clearing of blood in


each tube

Supernatant Xanthochromic (pink, orange or yellow) Clear

Presence of Clots No Yes, due to fibrinogen

Urinalysis and Other Body fluids 112


15. The principal mucin in synovial fluid is:

A. Hyaluronate
B. Albumin
C. Orosomucoid
D. Pepsin

ANSWER: A. Hyaluronate

Hyaluronic acid is the principal mucin in synovial fluid, and its role is lubrication of the
joints. A low hyaluronic acid leads to decreased viscosity of the synovial fluid and
inflammation. (Strasinger)

Urinalysis and Other Body fluids 113


16. Pleural transudates differ from pleural exudates in that transudates
have:

A. Protein values of >4g/dL


B. Specific gravity values of >1.020
C. LD values of >200 IU
D. Relatively low cell counts

ANSWER: D. Relatively low cell counts

Transudates are thin, watery effusions with low LD, low protein, and low cell counts.
Exudates are inflammatory or infectious with high LD, protein and WBC.

Urinalysis and Other Body fluids 114


Transudate vs Exudate
Transudate Exudate
Color Colorless Yellow-White (inflammation)
Red-Brown (hemorrhage)
Yellow-Brown (bilirubin)
Milky-Green (chylous fluid)
Turbidity Clear, Watery Cloudy, Viscous
Specific Gravity <1.015 >1.015
Protein <3 g/dl >3 g/dl
LD (Lactic Dehydrogenase) <200 IU >200 IU
Cell Count <1000/ul >1000/ul
Associated with Congestive Heart Failure, Infections and Malignancies
Changes in Hydrostatic
Pressure

Urinalysis and Other Body fluids 115


17. The most common genetic defect associated with cystic fibrosis is
called:

A. Delta-F508
B. Trisomy 21
C. Philadelphia Chromosome
D. Fragile X

ANSWER: A. Delta-F508

Delta-F508 is the deletion of phenylalanine at position 508 of the CFTR protein.


Trisomy 21 is a third copy of chromosome 21, associated with down syndrome. The
Philadelphia chromosome is a gene translocation associated with acute myelocytic
leukemia. Fragile X is a form of mental retardation caused by an increase in number of
nucleotide repeats.

Urinalysis and Other Body fluids 116


18. The tau isoform of transferrin is a carbohydrate deficient protein
found only in:

A. CSF
B. Sweat
C. Amniotic Fluid
D. Semen

ANSWER: A. CSF

Tau transferrin is found ONLY in CSF.

Urinalysis and Other Body fluids 117


19. Peritoneal lavage is used to:

A. Detect intra-abdominal bleeding in blunt injury


B. Dialyze patients with end stage renal disease (ESRD)
C. Replace ascites with Saline
D. Perform therapeutic thoracentesis

ANSWER: A. Detect intra-abdominal bleeding in blunt injury

A patient with blunt trauma, such as a car accident, may have internal bleeding.
Peritoneal lavage introduces a fixed volume of saline into the peritoneal cavity, and
withdraws an aliquot, RBC are counted. Counts greater than 100,000/uL indicate blun
trauma.

Urinalysis and Other Body fluids 118


20. Amniotic fluid is evaluated using a Liley graph and change in
absorbance at 450nm. What is being evaluated and why?

A. Bilirubin, which increases in HDN


B. AFP, which increases in spina bifida
C. HCG, which increases in down syndrome
D. Lamellar bodies, which increase with fetal lung maturity

ANSWER: A. Bilirubin, which increases in HDN

A fetus with hemolytic disease will have increased bilirubin in the amniotic fluid.
Bilirubin absorbs light at 450nm. The change in absorbance between the expected and
observed value is plotted on a Liley graph and used to assess the fetus. This is also
called a Delta-OD 450.

Urinalysis and Other Body fluids 119


120
III. CHEMISTRY

121
1. What does measuring the total iron binding capacity (TIBC)
represent?

A. Amount of free iron in serum


B. Circulating protein-bound iron
C. Amount of iron that transferrin can bind
D. Indirect measurement of iron stores

ANSWER: C. Amount of Iron that transferrin can bind

Total iron-binding capacity (TIBC) represents the amount of iron that circulating
transferrin could bind when fully saturated.

Chemistry 122
2. Which statement best describes the clinical utility of tests for
microalbuminuria?

A. Testing may detect early renal involvement in diabetes mellitus


B. Microalbuminuria refers to a specific subfraction of albumin found
only in persons with diabetic nephropathy
C. A positive test result indicates the presence of orthostatic
albuminuria
D. Testing should be part of the routine urinalysis

ANSWER: A. Testing may detect early renal involvement in DM

The microalbumin test is an assay for measuring urinary albumin concentration that has
an increased sensitivity (detection limit below 15 mg/dL), and is recommended for persons who
are at risk for chronic renal disease, especially persons with diabetes mellitus.

Chemistry 123
3. The glycated hemoglobin value represent the integrated values
of glucose concentration during the preceding:

A. 1-3 weeks
B. 4-5 weeks
C. 6-8 weeks
D. 16-20 weeks

ANSWER: C. 6-8 weeks

Total glycosylated hemoglobin levels in a hemolysate reflect the average blood glucose
of the past 2-3 months. Tietz 2006

Chemistry 124
4. What is the confirmatory method for measuring drugs of abuse?

A. HPLC
B. Enzyme-multiplied immunoassay technique (EMIT)
C. Gas chromatography with mass spectroscopy (GC-MS)
D. TLC

ANSWER: C. GC-MS

GC-MS determines the mass spectrum of the compounds eluting from the
analytic column. Each substance has a unique and characteristic spectrum of mass
fragments. This spectrum is compared to spectra in a library of standards to determine
the percent match. A match of greater than 95% is considered confirmatory.

Chemistry 125
5. A patient’s blood gas results are:
pH = 7.50 PCO2 = 55 mm Hg HCO3 – = 40 mmol/L

These results indicate:

A. Respiratory acidosis
B. Metabolic alkalosis
C. Respiratory alkalosis
D. Metabolic acidosis

ANSWER: B. Metabolic alkalosis

A pH above 7.45 corresponds with alkalosis. Both bicarbonate and PCO2 are elevated.
Bicarbonate is the conjugate base and is under metabolic (renal) control, while PCO2 is an acid
and is under respiratory control. Increased bicarbonate (but not increased CO2) results in
alkalosis; therefore, the classification is metabolic alkalosis, partially compensated by increased
PCO2.

Chemistry 126
Chemistry 127
6. Severe diarrhea causes?

A. Respiratory acidosis
B. Metabolic alkalosis
C. Respiratory alkalosis
D. Metabolic acidosis

ANSWER: D. Metabolic acidosis

Severe diarrhea = excessive loss of bicarbonate

Metabolic acidosis is decrease in CO2 content and PCO2 with a decrease pH

Chemistry 128
7. Anion gap is useful for quality control of laboratory results for:

A. Amino acids and proteins


B. Blood gas analyses
C. Sodium, potassium, chloride and total CO2
D. Calcium, Phosphorus and magnesium

ANSWER: C. Sodium, potassium, chloride and total CO2

Chemistry 129
8. The most specific test for acute pancreatitis is:

A. Acid phosphatase
B. Trypsin
C. Amylase
D. Lipase

ANSWER: D. Lipase

Elevated levels of lipase persists longer than amylase. Elevated levels of


lipase and amylase are seen in other intraabdominal conditions, but the frequency of
elevations is less with lipase than amylase.

Chemistry 130
9. The enzyme present in almost all tissues that may be separated
by electrophoresis into 5 components is:

A. Lipase
B. Transaminase
C. Creatine Kinase
D. Lactate dehydrogenase

ANSWER: D. Lactate dehydrogenase

Electrophoretically, using agarose or cellulose acetate medium, LD can be separated


into 5 isoenzymes: LD1-LD5.
CK and lipase have 3 isoenzymes.
AST has 2.

Chemistry 131
10. When myocardial infarction occurs, the first enzyme to become
elevated is:

A. CK
B. LD
C. AST
D. ALT

ANSWER: A. CK

After an AMI, CK activity increases 4-6 hours after symptoms, peaks at 12-24 hours
and returns to normal within 48-72 hours.

Chemistry 132
Remember “MTCAL”
Myoglobin
Troponin
CKMB
AST
LDH

Chemistry 133
Chemistry
11. The VLDL fraction primarily transports what substance?

A. Cholesterol
B. Chylomicron
C. Triglyceride
D. Phospholipid

ANSWER: C. Triglyceride (endogenous)

The VLDL fraction is primarily composed of triglycerides and lesser amounts of


cholesterol and phospholipids. Protein components of VLDL are mostly apolipoprotein
B-100 and apolipoprotein C. VLDL migrates electrophoretically in the prebeta region.

Chemistry 134
12. Name a commonly used precipitating reagent to separate HDL
cholesterol from other lipoprotein cholesterol fractions.

A. Zinc sulfate
B. Trichloroacetic acid
C. Heparin-manganese
D. Isopropanol

ANSWER: C. Heparin-Manganese

Either a dextran sulfate-magnesium chloride mixture or a heparin sulfate-


manganese chloride mixture may be used to precipitate the LDL and VLDL cholesterol
fractions. This allows the HDL cholesterol fraction to remain in the supernatant. An
aliquot of the supernatant may then be used in a total cholesterol procedure for the
quantification of the HDL cholesterol level.

Chemistry 135
13. The most important buffer pair in plasma is the:

A. Phosphate/biphosphate pair
B. Hemoglobin/imidazole pair
C. Bicarbonate/carbonic acid pair
D. Sulfate/bisulfate pair

ANSWER: C. Bicarbonate /carbonic acid pair

Most important buffer pair in plasma. Tietz 2006

Chemistry 136
14. A potassium level of 6.8 mEq/L (6.8 mmol/L) is obtained. Before
reporting the results, the first step the technologist should take is to:

A. Check the serum for hemolysis


B. Rerun the test
C. Check the age of the patient
D. Do nothing, simply report out the result

ANSWER: A. Check the serum for hemolysis

Effect of hemolysis in potassium. Tietz 2006

Chemistry 137
15. Calcium concentration is regulated by:

A. Insulin
B. Parathyroid hormone
C. Thyroxine
D. Vitamin C

ANSWER: B. Parathyroid hormone

Chemistry 138
16. Which of the following is most likely to be ordered in addition to
serum calcium to determine the cause of tetany?

A. Magnesium
B. Phosphate
C. Sodium
D. Vitamin D

ANSWER: A. Magnesium

Tetany and magnesium relationship. Tietz 2006

Chemistry 139
17. The primary function of serum albumin in the peripheral blood
is to:

A. Maintain colloidal osmotic pressure


B. Increase antibody production
C. Increase fibrinogen formation
D. Maintain blood viscosity

ANSWER: A. Maintain colloidal osmotic pressure

Physiological feature of albumin. Tietz 2006

Chemistry 140
18. The first step in analyzing a 24-hour urine specimen for quantitative
urine protein is:

A. Subculture the urine for bacteria


B. Add the appropriate preservative
C. Screen for albumin using a dipstick
D. Measure the total volume

ANSWER: D. Measure the total volume

Basic principle of lab procedure timed. Tietz 2006

Chemistry 141
19. The serum electrophoresis pattern shown below was obtained
on cellulose acetate at pH 8.6?
Identify the serum fraction on the left of the illustration.

A. Gamma globulin
B. Albumin
C. Alpha-1 globulin
D. Alpha-2 globulin

ANSWER: B. Albumin

Chemistry 142
Chemistry 143
20. The biuret reaction for the analysis of serum protein depends
on the number of:

A. Free amino groups


B. Free carboxyl groups
C. Peptide bonds
D. Tyrosine residues

ANSWER: C. Peptide bonds

Principle of biuret reaction. Tietz 2006

Chemistry 144
21. In electrophoresis of proteins, when the sample is placed in an
electric field connected to a buffer of pH 8.6, all of the proteins:

A. Have a positive charge


B. Have a negative charge
C. Are electrically neutral
D. Migrate toward the cathode

ANSWER: B. Have a negative charge

Principle of protein electrophoresis. Kaplan 2003

Chemistry 145
22. Refer to the illustration. The
serum electrophoresis pattern
is consistent with:

A. Cirrhosis
B. Acute inflammation
C. Monoclonal Gammopathy
D. Polyclonal Gammopathy

ANSWER: C. Monoclonal Gammopathy

Abnormal protein of a malignant cell.

❖ Albumin decreased
❖ Sharp peak in gamma region

Chemistry 146
23. Refer to the pattern.
This pattern is consistent with:

A. Liver Cirrhosis
B. Acute inflammation
C. Polyclonal gammopathy
D. Alpha-1 antitrypsin deficiency

ANSWER: A. Liver Cirrhosis

Abnormal liver function.

❖ Prominent beta – gamma bridging


(increased IgA)

Chemistry 147
24. Refer to the pattern. The serum
protein electrophoresis pattern is
consistent with?

A. Cirrhosis
B. Acute inflammation
C. Polyclonal gammopathy
D. Alpha-1 antitrypsin deficiency

ANSWER: B. Acute inflammation

❖ Increase of acute phase reactants alpha – 1 and


alpha – 2 (haptoglobin)

❖ Slight decrease of albumin and beta – 1 transferrin.

Chemistry 148
Chemistry 149
Condition Pattern

Polyclonal gammopathy Diffuse increase in gamma

Hypogammaglobulinemia Decreased gamma

Nephrotic Syndrome Decreased albumin


Increased alpha-2

Alpha-1-antitrypsin Deficiency Decreased alpha-1

Hemolyzed specimen Increased beta or unusual band between alpha-2


and beta

Plasma Extra band (fibrinogen) between beta and gamma

Chemistry 150
25. Analysis of CSF for oligoclonal bands is used to screen for which of
the following disease states?

A. Multiple myeloma
B. Myasthenia gravis
C. Multiple sclerosis
D. Von Willebrand disease

ANSWER: C. Multiple sclerosis

Clinical correlation, autoimmune immunoglobulin G.

Chemistry 151
Quantification by Densitometry
• AFTER ELECTROPHORESIS, A STAINED GEL IS PASSED
THROUGH THE OPTICAL SYSTEM OF A
DENSITOMETER.
• DENSITOMETER – SPECIAL SPECTROPHOTOMETER –
MEASURES LIGHT TRANSMITTED THROUGH A SOLID
SAMPLE
• CREATES AN ELECTROPHOREGRAM – VISUAL
DIAGRAM OR GRAPH OF THE SEPARATED BANDS
• MICROPROCESSOR EVALUATES THE AREA UNDER
EACH PEAK AND REPORTS EACH AS A PERCENTAGE
OF THE TOTAL SAMPLE

Chemistry
CSF protein electrophoresis
•Aids in diagnosis of Multiple Sclerosis
•Oligoclonal banding in the gamma region due to IgG produced in the CSF
oIgG NOT present in the serum

CSF,
MS Serum,
MS
Serum,
MS CSF, MS

CSF, Serum, normal


normal
CSF, normal
Protein electrophoresis Isoelectric focusing

Chemistry
26. Total-iron binding capacity measures the serum iron
transporting capacity of?

A. Hemoglobin
B. Ceruloplasmin
C. Transferrin
D. Ferritin

ANSWER: C. Transferrin

Measures transport function of transferrin.


Measure of Iron bound to transferrin.

Chemistry 154
27. An elevated serum iron with normal iron binding capacity is
most likely associated with?

A. Iron deficiency anemia


B. Renal damage
C. Sideroblastic anemia
D. Septicemia

ANSWER: C. Sideroblastic anemia

Ineffective erythropoiesis, high iron turnover.

Chemistry 155
Good to know

Chemistry 156
28. A serum sample demonstrates an elevated result when tested
with Jaffe reaction. This indicates:

A. Prolonged hypothermia
B. Renal functional impairment
C. Pregnancy
D. Arrhythmia

ANSWER: B. Renal functional impairment

Jaffe reaction = creatinine measurement = kidney function test

Chemistry 157
29. The principle excretory form of nitrogen is:

A. Amino acids
B. Creatinine
C. Urea
D. Uric acid

ANSWER: C. Urea

Metabolic product of protein catabolism

Chemistry 158
30. The hemoglobin that is resistant to alkali (KOH) denaturation is:

A. A
B. A2
C. C
D. F

ANSWER: D. Hemoglobin F

Physical-chemical property of Hgb F

Chemistry 159
31. Which of the following would cause an increase in only the
unconjugated bilirubin?

A. Hemolytic anemia
B. Obstructive jaundice
C. Hepatitis
D. Hepatic cirrhosis

ANSWER: A. Hemolytic anemia


Conjugated bilirubin increases as a result of obstructive processes within the liver or
biliary system or from failure of the enterohepatic circulation. Hemolytic anemia (prehepatic
jaundice) presents a greater bilirubin load to a normal liver, resulting in increased bilirubin
excretion. When the rate of bilirubin formation exceeds the rate of excretion, the unconjugated
bilirubin rises.

Chemistry 160
• Total Bilirubin: 0.2-1 mg/dL
• Conjugated Bilirubin (direct): <0.2 mg/dL
• Unconjugated Bilirubin (indirect): <0.8 mg/dL

Chemistry 161
32. Which form of hyperbilirubinemia is caused by an inherited absence of UDP-
glucuronyl transferase?

A. Gilbert’s syndrome
B. Rotor syndrome
C. Crigler–Najjar syndrome
D. Dubin–Johnson syndrome

ANSWER: C. Crigler-Najjar syndrome

Crigler–Najjar syndrome is a rare condition that occurs in two forms. Type 1 is


inherited as an autosomal recessive trait and causes a total deficiency of UDP-
glucuronyl transferase. Life expectancy is less than 1 year. Type 2 is an autosomal
dominant trait and is characterized by lesser jaundice and usually the absence of
kernicterus.

Chemistry 162
Serum Conjugated Serum Unconjugated Bilirubin
Bilirubin

Excessive erythrocyte destruction Increased


Prehepatic
Gilbert Syndrome Defect in hepatocytes ability to Mild increase
Hepatic take up bilirubin

Crigler-Najjar disease Partial or complete deficiency of Mild to extreme Increase


UDP-glucuronyltransferase

Dubin-Johnson syndrome Impaired transport in the Increased


hepatocyte to excrete conju
bilirubin from microsomal region
to bile canaliculi

Neonatal Physiological UDP-glucuronyltransferase is low Increased


jaundice at birth

Intrahepatic cholestasis
Intrahepatic cholestasis Hepatocyte injury, cirrhosis, bile
duct injury (rotor syndrome),
neoplasms

Obstruction blocks flow of bile to Significantly increased Increased


Post Hepatic intestines. Extrahepatic
cholestasis

Chemistry 163
33. The term R4S means that:
A. Four consecutive controls are greater than ±1 standard deviation from the
mean

B. Two controls in the same run are greater than 4s units apart

C. Two consecutive controls in the same run are each greater than ±4s from the
mean

D. There is a shift above the mean for four consecutive controls

ANSWER: B. Two controls in the same run are greater than 4s units apart

The R4s rule is applied to two control levels within the same run. The rule is violated
when the algebraic difference between them (level 1 –
level 2) exceeds 4s. The rule is never applied across different runs. The R4s rule detects
random error (error due to poor precision).

Chemistry 164
34. Select the correct order of Hgb migration on citrate agar at pH 6.2.

A. – F→S→C→A +
B. – F→A→S→C +
C. – A→S→F→C +
D. – A→C→S→F +

ANSWER: B. – F→A→S→C +

In an acid buffer, the hemoglobins are expected to migrate to the cathode.


Hemoglobin A is the slowest because it has the weakest net positive charge.
However, Hgb C and Hgb S bind to sulfated pectins in the agar gel, forming a complex that is
negatively charged causing them to migrate toward the anode. Hgb C migrates furthest toward
the anode, followed by Hgb S. Hgb F migrates furthest toward the cathode.

Chemistry 165
35. Which of the following diseases is caused by a deficiency of sphingomyelinase?

A. Gaucher disease
B. Fabry disease
C. Niemann–Pick disease
D. Tay–Sachs disease

ANSWER: C. Niemann-Pick Disease

The diseases mentioned result from inborn errors of lipid metabolism


(lipidoses) caused by deficiency of an enzyme needed for lipid degradation. Specific
lipids accumulate in the lysosomes. Niemann–Pick disease results from a deficiency of
sphingomyelinase; Gaucher disease from β-glucocerebrosidase; Fabry disease (sex
linked) from α-galactosidase A; and Tay–Sachs from N acetylglucosaminidase A.

Chemistry 166
36. Which isoenzyme of ALP migrates farthest toward the anode when
electrophoresed at pH 8.6?

A. Placental
B. Bone
C. Liver
D. Intestinal

ANSWER: C. Liver

Liver ALP isoenzymes migrate farthest toward the anode, but fast and slow variants occur. The
slow liver ALP band is difficult to distinguish from placental and bone ALP. The order from
cathode to anode is:
– Renal→Intestinal→Bone→Placental→Liver +

Improved separation of bone and liver isoenzymes can be achieved by incubating the
serum with neuraminidase prior to electrophoresis. The enzyme reduces the sialic content of the
bone isoenzyme, causing it to migrate at a slower rate.

Chemistry 167
37. Hyperparathyroidism is most consistently associated with:

A. Hypocalcemia
B. Hypocalciuria
C. Hypophosphatemia
D. Metabolic alkalosis

ANSWER: C. Hypophosphatemia

Hyperparathyroidism causes increased resorption of calcium and decreased renal


retention of phosphate.
Increased serum calcium leads to increased urinary excretion. The distal collecting
tubule of the nephron reabsorbs less bicarbonate as well as phosphate, resulting in acidosis.

Chemistry 168
38. Select the most appropriate single screening test for thyroid disease.

A. Free thyroxine index


B. Total T3 assay
C. Total T4
D. TSH assay

ANSWER: D. TSH Assay

TSH is produced by the anterior pituitary in response to low levels of free T4 or


T3. A normal TSH rules out thyroid disease. TSH is low in primary hyperthyroidism and
high in primary hypothyroidism.

Chemistry 169
MUST KNOW!
Thyroid State Disease Causes Symptoms

Hashimoto’s thyroiditis (Most common) autoimmune disorder Goiter may or may not occur
Low free T4
Elevated TSH
Hypothyroidism Treat with T4 (6-4 weeks to achieve steady state)

Cretinism Congenital
Myxedema
Grave’s Disease (thyrotoxicosis) autoimmune disorder Diffuse toxic goiter
B lymphs make antibodies to TSH receptor
High T3, T4, RT3U, FT4, FT4 Index
Low TSH

Toxic Goiter (uni or multinodular) Marked elevation of T3 (T3 thyrotoxicosis)


Treat with surgery or methimazole

Hyperthyroidism Subacute Thyroiditis Transient release of stored T4 and T3


Spontaneously Resolves

Thyroid Storm (Thyrotoxic Crisis) Life threatening syndrome


Potassium iodide to block T4 release from gland
Propylthiouracil to block T4 synthesis

Nontoxic Goiter Iodide deficiency in the diet and Thyroid Enlargement with no excess T4
hashimoto's thyroiditis Enlargement due to excess TSH
Goitrogens in diet

Chemistry
LAB VALUES IN Thyroid dysfunction
TSH FT4 T3

Primary hypothyroidism Increased Decreased -

Central hypothyroidism Decreased Decreased -

Subclinical hypothyroidism Increased Normal -

Primary hyperthyroidism Decreased Increased Increased

T3 toxicosis Decreased Normal Increased

Subclinical hyperthyroidism Decreased Normal Normal

Chemistry
39. When should blood samples for trough drug levels be collected?

A. 30 minutes after peak levels


B. 45 minutes before the next dose
C. 1–2 hours after the last dose
D. Immediately before the next dose is given

ANSWER: D. Immediately before the next dose is given

The trough concentration of a drug is the lowest concentration obtained in the dosing
interval. This occurs immediately before the absorption of the next dose given. Trough
levels are usually collected just before the next dose is given.

Chemistry 172
When is the best time to draw sample for the trough
measurement of Phenytoin? (Recall September 2021)

A.within 2-4 hrs post IV load


B.4-8 hrs after
C.Immediately prior to next dose
D.2-4 hrs after

ANSWER: C. Immediately prior to next dose

Chemistry 173
40. Which of the following assays is recommended as a screening test for colorectal
cancer in persons over 50 years old?

A. CEA
B. AFP
C. Occult blood
D. Fecal trypsin

ANSWER: C. Occult blood

Bleeding in the gastrointestinal tract occurs during the early stages of colorectal cancer
when treatment can be most effective. Although occult blood can be caused by many other GI
problems, it is not associated with benign polyps and has a sensitivity of over 80% for detection
of colorectal cancer. CEA is elevated in less than 60% of such cases. AFP is elevated in only
about 5% of colon cancers. Fecal trypsin is not a marker for colorectal cancer, but α1-antitrypsin
is present in the stool in a majority of malignant colon tumors owing to intestinal protein loss.

Chemistry 174
41. Which condition is associated with the lowest percent saturation of transferrin?

A. Hemochromatosis
B. Anemia of chronic infection
C. Iron deficiency anemia
D. Noniron deficiency anemia

ANSWER: C. Iron deficiency anemia

Percent saturation = Serum Fe × 100/TIBC. Normally, transferrin is one-third


saturated with iron. In iron deficiency states, the serum iron falls but transferrin rises.
This causes the numerator and denominator to move in opposite directions, resulting
in very low percent saturation (about 10%). The opposite occurs in hemochromatosis
and sideroblastic anemia, resulting in an increased percent saturation.

Chemistry 175
42. Which of the following results falls within the diagnostic criteria for diabetes mellitus?

A. Fasting plasma glucose of 120 mg/dL


B. Two-hour postprandial plasma glucose of 160 mg/dL
C. Two-hour plasma glucose of 180 mg/dL following a 75 g oral glucose challenge
D. Random plasma glucose of 250 mg/dL and presence of symptoms

ANSWER: D. Random plasma glucose of 250 mg/dL and presence of symptoms

The American Diabetes Association recommends the following criteria for diagnosing diabetes
mellitus: fasting glucose ≥ 126 mg/dL, casual (random) glucose ≥ 200 mg/dL in the presence of
symptoms (polyuria, increased thirst, weight loss), glucose ≥ 200 mg/dL at 2 hours after an oral dose of
75 g of glucose, and hemoglobin A1c ≥ 6.5%. A diagnosis of diabetes mellitus is indicated if any one or
combination of these four criteria is met on more than a single testing event. The fasting plasma glucose
test requires at least 8 hours with no food or drink except water. The 2-hour postloading test should be
conducted according to the oral glucose tolerance guidelines currently recommended by the World
Health Organization.

Chemistry 176
Chemistry 177
Chemistry 178
43. What is the recommended cutoff for the early detection of chronic kidney disease in
diabetics using the test for microalbuminuria?

A. >30 mg/g creatinine


B. >80 mg/g creatinine
C. >200 mg/g creatinine
D. >80 mg/L

ANSWER: A. >30 mg/g creatinine

Microalbuminuria is the excretion of small quantities of albumin in the urine. In diabetics,


excretion of albumin that is within allowable limits for healthy persons may signal the onset of chronic
kidney disease. The term microalbuminuria is defined as albumin excretion ≥ 30 mg/g creatinine but
≤ 300 mg/g creatinine. The use of the albumin to creatinine ratio is preferred to measures of albumin
excretory rate (μg/min) because the latter is subject to error associated with timed specimen
collection. ADA recommends the test be done annually for all type 2 diabetics and type 1 diabetics
who have had the disease for > 5 years.

Chemistry 179
44. Which of the following is the reference method for measuring serum glucose?

A. Somogyi–Nelson
B. Hexokinase
C. Glucose oxidase
D. Glucose dehydrogenase

ANSWER: B. Hexokinase

The hexokinase method is considered more accurate than glucose oxidase


methods because the coupling reaction using glucose-6-phosphate dehydrogenase
(G-6-PD) is highly specific. The hexokinase method may be done on serum or plasma
collected using heparin, EDTA, fluoride, oxalate, or citrate. The method can also be
used for urine, cerebrospinal fluid, and serous fluids.

Chemistry 180
45. Adrenal cushing syndrome causes?

a. ↑ACTH ↑cortisol
b. ↑ACTH ↓cortisol
c. ↓ACTH ↓cortisol
d. ↓ACTH ↑cortisol

ANSWER: A. ↑ACTH ↑cortisol

Cushing syndrome increases ACTH and Cortisol

Chemistry 181
46. Which of the following statements about amniotic fluid bilirubin measured by scanning
spectrophotometry is true?

A. The 410-nm peak is due to hemoglobin and the 450-nm peak is due to bilirubin
B. Baseline correction is not required if a scanning spectrophotometer is used
C. Chloroform extraction is necessary only when meconium is present
D. In normal amniotic fluid, bilirubin increases with gestational age

ANSWER: A. The 410-nm peak is due to hemoglobin and the 450-nm peak is due to bilirubin

Amniotic fluid bilirubin reflects the extent of fetal RBC destruction in cases of hemolytic disease of the
newborn (HDN). It is measured by scanning the fluid from 350 to 600 nm, then drawing a baseline using the
points at 365 nm and 550 nm. The delta absorbance (ΔA) of hemoglobin at 410 nm and bilirubin at 450 nm are
determined by subtracting the absorbance of the baseline from the respective peaks.

Chemistry 182
47. The ion-selective membrane used to measure potassium is made of:

A. High-borosilicate glass membrane


B. Silver Chloride
C. Valinomycin gel
D. Calomel

ANSWER: C. Valinomcyin

Valinomycin is an antibiotic with a highly selective reversible-binding affinity for potassium


ions.

Chemistry 183
REMEMBER!

Ion-Selective Electrodes

Sodium: Glass-electrode
Potassium: Valinomycin
Chloride: Solid state electrodes containing AgCl (Silver Chloride)

Chemistry 184
TIPS!

Study formulas, familiarize normal values, electrophoresis patterns,


tumor markers, enzymes and QCs.

ANION GAP

CREATININE CLEARANCE

Chemistry 185
Reference Intervals
Electrolyte
(NHANES III)

Na+ 136 to 146 mmol/L

K+ 3.4 to 4.7 mmol/L

Cl- 98 to 111 mmol/L

CO2 21 to 35 mmol/L

Chemistry
RIDDLE TIME!

What kind of tree can fit into


your hand?

A palm tree

187
IV. HEMATOLOGY

218
1. If CSF tubes numbered #1, #2 and #3 CANNOT be analyzed within one hour, the correct
procedure for the microbiology sample tube is to:

A. Refrigerate it
B. Freeze it
C. Leave it at room temperature
D. Incubate in a 56-degree water bath

ANSWER: C. Leave it at room temperature

You must leave the tube at room temperature since refrigerating, freezing, or incubating the
specimen at 56 degrees will destroy Neisseria meningitides (if present). The sample should not be
thrown away.
Tube #1 is used for chemistry or serological testing and can be stored frozen.
Tube #2 is used for microbiology testing and should remain at room temperature.
Tube #3 is used for cell count in the hematology department and should be stored in the refrigerator.

Hematology 219
2. A 41-year-old male arrived at the clinic with complaints of fatigue, malaise, and overall discomfort. Upon
testing, the patient's CBC results were notably abnormal with 30% blasts present along with anemia and
thrombocytopenia. Bone marrow and cytochemical staining were subsequently performed. The cytochemical
stains noted negative reactions to Myeloperoxidase and Sudan Black B but a positive reaction to alpha-
naphthyl acetate. Which of the following conditions is most likely seen?

A. Acute Erythroleukemia
B. Acute Monocytic Leukemia
C. Acute Promyelocytic Leukemia
D. Acute Lymphocytic Leukemia

ANSWER: B. Acute Monocytic Leukemia (AML)

Acute Monocytic Leukemia is most likely present due to the positive reaction of alpha-naphthyl acetate stain.
This is a type of nonspecific esterase stains which are known to positively identify monoblastic cells. Specific esterase
stains would stain myelocytic cells positive.
Both Acute Erythroleukemia and Acute Lymphocytic Leukemia would stain negative with all three stains
mentioned in the case.
Acute Promyelocytic Leukemia would stain positive with Myeloperoxidase and Sudan Black B, but would stain
negatively with alpha-naphthyl acetate.

Hematology 220
Good to know!
Cytochemical Reactions in Acute Leukemia
Cellular Element
Cytochemical Reaction Blasts Identified
Stained
Neutrophil primary Myeloblasts strong positive; monoblasts
Myeloperoxidase (MPO)
granules faint positive
Myeloblasts strong positive; monoblasts
Sudan Black B (SBB) Phospholipids
faint positive
Specific esterase Cellular enzyme Myeloblasts strong positive
Nonspecific esterase
Cellular enzyme Monoblasts strong positive
(NSE)

Variable, coarse or block-like positivity


often seen in
lymphoblasts and pronormoblasts,
Glycogen and related
Periodic acid-Schiff myeloblasts usually
substances
negative although faint diffuse reaction
may
occasionally be seen

Hematology
3. What are the globular inclusions in this cell?

A. Bacteria
B. Fat Droplets
C. Vacuoles
D. Russell bodies

ANSWER: D. Russell Bodies

The prominent cell is a Mott cell on a peripheral blood smear from a patient with
multiple myeloma.
The globular inclusions are immunoglobulin accumulations (Russell bodies)
representing the accumulation of excessive mucopolysaccharides and globulins within
endoplasmic reticulum.

Hematology 222
4. Which of the following statements regarding the peripheral blood smear shown in the
photomicrograph is TRUE?

A. This area is acceptable for performing a manual differential


B. This area is unacceptable for scanning for blast cells, parasites, fibrin strands, and platelet
clumps
C. This area is unacceptable for evaluating red blood cell morphology because it is too thin
D. This area is acceptable for evaluating red blood cell morphology

Hematology 223
#4 ANSWER: C. This area is unacceptable for evaluating red blood cell morphology because it is too thin

This photomicrograph represents an area of the peripheral blood smear that is too thin for performing
a white blood cell differential, as well as evaluating red blood cell, white blood cell, or platelet morphology. The
red blood cells appear flat, distorted, and lack central pallor. This is typical of red blood cells in the feathered
edge.
The white blood cell differential and evaluation of red blood cell, white blood cell, and platelet
morphology must be performed in the monolayer, also known as the critical area of the smear. The monolayer
is the area of the smear where red blood cells are evenly distributed, barely overlap, and have nice central
pallor.
The area shown is acceptable for scanning for blast cells, parasites, fibrin strands, and platelet clumps at
a low magnification, and omitting this step as part of peripheral blood smear evaluation is a common error in
the hematology laboratory.

Hematology 224
5. Refer to the illustration

Which curve represents the production of alpha polypeptide chains of hemoglobin?

A. A
B. B
C. C
D. D

Hematology 225
Hemoglobin Production

#4 ANSWER: B.

Curve B represents the time line


of production of Hgb alpha chains

Curve A: Epsilon
Curve C: Gamma
Curve D: Beta
Curve E: Delta

Hematology
6. The characteristic morphologic feature of lead poisoning is:

A. Macrocytosis
B. Target cells (codocytes)
C. Basophilic stippling
D. Rouleax formation

ANSWER: C. Basophilic stippling

Hematology 227
7. The characteristic peripheral blood morphologic feature in
multiple myeloma is:

A. Cytotoxic T cells
B. Rouleax formation
C. Spherocytosis
D. Macrocytosis

ANSWER: B. Rouleax formation


The peripheral smear allows microscopic examination of blood cells. The most
characteristic finding in multiple myeloma is rouleax formation of the red cells.

Hematology 228
8. Hemoglobin H disease results from:

A. Absence of 3 of 4 alpha genes


B. Absence of 2 of 4 alpha genes
C. Absence of 1 of 1 alpha genes
D. Absence of all 4 alpha genes

ANSWER: A. Absence of 3 of 4 alpha genes

Hemoglobin H disease occurs when 3 of 4 alpha genes are deleted

Hematology 229
Hemoglobin Structure

Hematology
REVIEW!
αThalassemia
Large deletions involving the α1 and or α2 globin genes are the predominant defect.
The extent of decreased production of α chain depends on:
◦ The specific mutation
◦ The number of α globin chain affected
◦ Whether the affected α globin gene is α1 or α2. The α2 globin genes produces 75% of
αchains in the normal RBCs.

Hematology
4 Clinical Syndromes of
α Thalassemia
1.) Silent Carrier State 3.) Hemoglobin H disease (α
◦ Deletion of one αglobin gene, leaving three Thalassemia Intermedia)
functional ones (-α/αα) is the major cause of ◦ Deletion of three genes which is the
silent carrier state. major cause of Hemoglobin H disease (--
2.) α Thalassemia Minor (α Thalassemia /-α)
trait) ◦ It is characterized by the accumulation of
◦ Deletion of two α globin genes excess unpaired β chains that form
◦ It exists in 2 forms: tetramers of Hb H in adults.
◦ Homozygous (-α/-α)
◦ Heterozygous (--/αα)

Hematology
Hemoglobin H disease
(α Thalassemia Intermedia )

RBC from a patient with Hemoglobin H disease incubated with brilliant cresyl blue

Hematology
9. Cells that exhibit a positive stain with acid phosphatase and are not
inhibited with tartaric acid are characteristically seen in:

A. Infectious mononucleosis
B. Infectious lymphocytosis
C. Hairy cell leukemia
D. T-cell acute lymphoblastic leukemia

ANSWER: C. Hairy cell leukemia

A variable number of malignant cells in hairy cell leukemia (HCL) will stain
positive with tartrate-resistant acid phosphatase (TRAP+). Although this cytochemical
reaction is fairly specific for HCL, TRAP activity has occasionally been reported in B-
cell and rarely T-cell leukemia.

Hematology 234
10. Mean cell volume (MCV) is calculated using the following formula:

A. (Hgb ÷ RBC) × 10
B. (Hct ÷ RBC) × 10
C. (Hct ÷ Hgb) × 100
D. (Hgb ÷ RBC) × 100

ANSWER: B. (Hct ÷ RBC) × 10

MCV is the average “volume” of the red cells. This is obtained by dividing the
Hct or packed cell volume (PCV) by the red blood cell (RBC) count in millions per
microliter of blood and multiplying by 10. The MCV is expressed in cubic microns
(μm3) or femtoliters (fL).

Hematology 235
11. Which of the following is the formula for MCHC?

A. (Hgb x 100) / Hct


B. Hgb / RBC
C. RBC / Hct
D. (Hct x 1000) / RBC

ANSWER: A. (Hgb x 100) / Hct

Hematology 236
Hematology 237
12. The Prussian blue staining of peripheral blood identifies?

A. Howell-Jolly bodies
B. Siderotic granules
C. Reticulocytes
D. Basophilic stippling

ANSWER: B. Siderotic granules

Sideroblasts and siderocytes may be identified with Perl Prussian blue iron stain.

Hematology 238
13. Inclusions in the cytoplasm of neutrophils as shown in the
figure below are known as:

A. Auer bodies
B. Howell-Jolly bodies
C. Heinz Bodies
D. Dohle bodies

ANSWER: D. Dohle bodies

Hematology 239
Remember all the RBC inclusions. Refer to the table below from Clinical laboratory Science Review: A Bottom Line Approach 5 th Edition

Hematology 240
14. Which of the following results from decreased synthesis of globin
chains?

A. Beta-thalassemia
B. Hemoglobin C disease
C. Hemoglobin M
D. Sickle Cell disease

ANSWER: A. Beta-thalassemia

All other options result from structural abnormalities

Hematology 241
15. A peripheral blood smear stained with Prussian blue demonstrates
siderocytes. On a wright stained smear, what would be expected?

A. Basophilic stippling
B. Howell Jolly bodies
C. Heinz Bodies
D. Pappenheimer bodies

ANSWER: D. Pappenheimer bodies

Siderotic granules are composed of iron and on a wright stained smear appear
as pappenheimer bodies within the red cell. They are frequently seen in sideroblastic
anemia, alcoholism, thalassemia and some preleukemic states.

Hematology 242
16. Which of the following statements is true?

A. Reticulocytes are demonstrable


B. Toxic granulation is present
C. The cell in the center is a basophilic normoblast
D. The large cell on the left is a monocyte

ANSWER:

D. The large cell on the left is a monocyte

Hematology 243
17. The small nucleated cell seen in the lower left corner is a:

A. Polychromatophilic normoblast
B. Mature lymphocyte
C. Plasma cell
D. Lymphoblast

ANSWER:
B. Mature lymphocyte

Hematology 244
18. The cells seen in the image below are consistent with:

A. Chronic myelogenous leukemia


B. Infectious mononucleosis
C. Acute lymphocytic leukemia
D. Sezary syndrome

ANSWER:
B. Infectious Mononucleosis

Hematology 245
19. The large cell indicated by the arrow in the image below is a:

A. Myeloblast
B. Promyelocyte
C. Myelocyte
D. Metamyelocyte

ANSWER:

C. Myelocyte

Hematology 246
Myelocyte

Hematology
Metamyelocyte

Hematology
20. A useful chemical test for the diagnosis of hairy cell leukemia is the:

A. Peroxidase test
B. Sudan black test
C. Periodic acid-Schiff test
D. Tartrate-resistant acid phosphatase test

ANSWER: D. TRAP

TRAP stain positivity in hair cell leukemia.

Hematology 249
21. The leukocyte alkaline phosphatase (LAP) stain on a patient gives the following
results ?
10(0) 48(1+) 38(2+) 3(3+) 1(4+)
Calculate the LAP score.
A. 100
B. 117
C. 137
D. 252

ANSWER: C. 137

One hundred mature neutrophils are counted and scored. The LAP score is
calculated as: (the number of 1+ cells × 1) + (2+ cells × 2) + (3+ cells × 3) + (4+ cells × 4).

That is, 48 + 76 + 9 + 4 = 137. The reference range is approximately 20–130.

Hematology 250
22. Review the scatterplot of white blood cells shown. Which section of
the scatterplot denotes the number of monocytes?

A. A
B. B
C. C
D. D

ANSWER: A. Section A

White blood cell identification is facilitated by analysis of the impedance, conductance,


and light-scattering properties of the WBCs. The scatterplot represents the relationship between
volume (x axis) and light scatter (y axis). Monocytes account for the dots in section A,
neutrophils are represented in section B, eosinophils in section C, and lymphocytes are
denoted in section D.

Hematology 251
SCATTERPLOT

The prepared sample is analyzed using fluorescence


flow cytometry. The measurement signals related to side
scatter (SSC) and side fluorescence (SFL) are analyzed and
depicted in a scattergram. Cells with similar cytochemical The scattergram shows a normal cell distribution as
properties fall within the same area in the scattergram and can measured in the XN-Series analysers
be separated using an advanced software algorithm. Source: sysmex website
Hematology 252
SCATTERPLOT

EOSINOPHIL
NEUTROPHILS
MONOCYTES

LYMPHOCYTES

The scattergram shows a normal cell distribution as


measured in the XN-Series analysers
Source: sysmex website
Hematology 253
SCATTERPLOT with abnormal WBCs

Hematology 254
HISTOGRAM

Hematology 255
RED CELL HISTOGRAM
❖ It represents the relation between red cells size and the number
❖ If RBCs are larger than normal = shift to right
❖ If RBCs are smaller than normal = shift to left
❖ If curve is bimodal = dimorphic RBCs (2 populations)

Hematology 256
PLATELETS HISTOGRAM
❖ Platelet size is plotted against the number
❖ 2 parameters can be obtained: MPV and PDW
❖ If MPV >10 (Sickle cell anemia, splenectomy, ITP)
❖ If MPV <7 (Aplastic anemia, megaloblastic anemia, chemotherapy,
heterozygous thalassemia)

Hematology 257
WBC HISTOGRAM
35-90 fL: lymphocytes
90-160 fL: mononuclears (monocytes, blasts, immature granulocytes and
reactive lymphocytes)
160-450 fL: are granulocytes (basophils, eosinophils and neutrophils)

Hematology 258
23. Interpret the histogram

A. Neutrophilia
B. Lymphocytosis
C. Monocytosis
D. Eosinophilia

ANSWER:

C. MONOCYTOSIS

Monocytes are in 90-160 fL

Hematology 259
OTHER ABNORMAL
WBC histograms

Hematology 260
OTHER ABNORMAL WBC histograms

Cellular Interference Blasts

Hematology 261
ABNORMAL
RED CELL
HISTOGRAMS

(A) Normal histogram, (B) Microcytosis, iron deficiency anemia, (C) Microcytosis, beta thal trait, (D) Macrocytosis
with normal RDW, (E) Macrocytosis, megaloblastic anemia, (F) Cold agglutination, (G) Sideroblastic anemia, (H)
Beta thalassemia major, (I) Pyropoikilocytosis, (J) Reticulocytosis, (K) Post-iron therapy, (L) Post-iron therapy.
Hematology 262
ABNORMAL RED CELL HISTOGRAMS

RBC Fragments, dimorphic RBC


RBC Agglutination and elevated RDW

Dimorphic RBC
Macrocytic and elevated RDW

Hematology 263
ABNORMAL PLATELET HISTOGRAMS

Small platelets Giant platelets

Hematology 264
25. Refer to the following scatterplot, histograms, and
automated values on a 45-year-old man. What follow-up
verification procedure is indicated before releasing these
results?

a. Redraw blood sample using a sodium citrate tube;


multiply PLTs × 1.11
b. Dilute the WBCs 1:10; multiply × 10
c. Perform plasma blank Hgb to correct for lipemia
d. Warm specimen at 37°C for 15 minutes; rerun
specimen

ANSWER: A. Redraw blood sample using a sodium


citrate tube; multiply PLTs × 1.11

The platelet clumping phenomenon is often


induced in vitro by the anticoagulant EDTA. Redrawing a
sample from the patient using a sodium citrate tube
usually corrects this phenomenon and allows accurate
platelet enumeration.

Hematology 265
26. Which of the following is associated with pseudo-Pelger-Huet
anomaly?

A. Aplastic anemia
B. Iron deficiency anemia
C. Myelogenous leukemia
D. Chediak-Higashi syndrome

ANSWER: C. Myelogenous leukemia

Differentiates leukemia with classic granulocyte anomaly. (McPherson 2007)

Hematology 266
27. Which of the following conditions may produce spherocytes in a peripheral smear?

A. Pelger–Huët anomaly
B. Pernicious anemia
C. Autoimmune hemolytic anemia
D. Sideroblastic anemia

ANSWER: C. Autoimmune Hemolytic Anemia

Spherocytes are produced in autoimmune hemolytic anemia. Spherocytes may be produced


by one of three mechanisms. First, they are a natural morphological phase of normal red cell
senescence. Second, they are produced when the cell surface-to-volume ratio is decreased, as seen
in hereditary spherocytosis. And third, they may be produced as a result of antibody coating of the
red cells. As the antibody-coated red cells travel through the spleen, the antibodies and portions of
the red cell membrane are removed by macrophages.

The membrane repairs itself; hence, the red cell’s morphology changes from a biconcave
disk to a spherocyte.

Hematology 267
28. Which of the following is the primary Hgb in patients with thalassemia major?

A. Hgb D
B. Hgb A
C. Hgb C
D. Hgb F

ANSWER: D. Hgb F

Patients with thalassemia major are unable to synthesize the β-chain; hence,
little or no Hgb A is produced. However, γ-chains continue to be synthesized and lead
to variable elevations of Hgb F in these patients.

Hematology 268
29. In essential thrombocythemia, the platelets are:

A. Increased in number and functionally abnormal


B. Normal in number and functionally abnormal
C. Decreased in number and functional
D. Decreased in number and functionally abnormal

ANSWER: A. Increased in number and functionally abnormal

In essential thrombocythemia, the platelet count is extremely elevated. These


platelets are abnormal in function, leading to both bleeding and thrombotic diathesis.

Hematology 269
30. What would be the most likely designation by the WHO for the FAB AML M3 by the
French–American–British classification?

A. AML with t(15;17)


B. AML with mixed lineage
C. AML with t(8;21)
D. AML with inv(16)

ANSWER: A. AML with t(15;17)

AML with t(15;17) is classified under the category of AML with Recurrent Genetic
Abnormalities by the WHO. Acute promyelocytic leukemia (PML; known as M3 under the FAB
system) is composed of abnormal promyelocytes with heavy granulation, sometimes obscuring
the nucleus, and abundant cytoplasm. Acute promyelocytic leukemia (APL) contains a
translocation that results in the fusion of a transcription factor called PML on chromosome 15
with the alpha (α)-retinoic acid receptor gene (RARα) on chromosome 17.

Hematology 270
Case study was given with the main key words: Translocation
of chromosome 15 & 17, DIC, blood film seen

ANSWER: M3 Acute Promyelocytic Leukemia


Recall September 2021

Hematology 271
31. In the French-American-British (FAB) Classification, acute
lymphocytic leukemia is divided into groups according to:

A. Prognosis
B. Immunology
C. Cytochemistry
D. Morphology

ANSWER: D. Morphology

Differentiation between morphologic basis used by FAB vs WHO classification,


which focuses on cytogenetic and molecular findings.

Hematology 272
32. Chronic lymphocytic leukemia (CLL) is defined as a(n):

A. Malignancy of the thymus


B. Accumulation of prolymphocytes
C. Accumulation of hairy cells in the spleen
D. Accumulation of monoclonal B cells with a block in cell maturation

ANSWER: D. Accumulation of monoclonal B cells with a block in cell maturation

Hematology 273
GOOD TO KNOW!

Hematology 274
33. What RBC inclusion can be seen on blood smear of a child who
accidentally ingested moth balls? or (naphthalene balls)

Answer:

Heinz Bodies

Hematology 275
34. Which of the following cells release histamine/heparin?

Answer:
Eosinophil and Basophil

Hematology 276
35. Identify the RBC inclusion:

Answer:
Howell Jolly Body

Hematology 277
278
MUST KNOW!
Another RECALL:
A peripheral blood picture with acanthocyte and
teardrop cells.

Asked what is the diagnosis?

Acanthocyte: Abetalipoproteinemia, severe


liver disease

Teardrop cells: Extramedullary, hematopoiesis,


thalassemias, pernicious anemias

Hematology 279
Acanthocytes

Echinocytes

Hematology
Teardrop Cells - Dacrocytes

Hematology
Stomatocytes

Hematology
HELPFUL PHOTOS
From: Cellavision Atlas

Hematology 283
HELPFUL PHOTOS
From: Cellavision Atlas

Hematology 284
HELPFUL PHOTOS
From: Cellavision Atlas

Other source of photos for FREE: http://www.bloodline.net/imageatlas/


Hematology 285
Hematology 286
Erythropoiesis

Hematology
Name Nucleus Cytoplasm
Cell image Cell size (µ) N:C ratio
ASCP Synonym Shape Color N. Chromatin Nucleoli Color Granules

Pro- Erythroblast
Reddish-
Round Close mesh network forming
Rubriblast 14-24 Blue 0-2 8:1-6:1 Lots of RNA so; dark blue None
Primitive a reticular appearance
Pro- Normoblast

Basophilic normo Reddish- Coarsens. Increased


Prorubricyte 12-17 Round Disappear 6:1-4:1 Blue with hints of pink None
(erythro)blast Blue granularity

Polychromatophilic
Thickened and irregularly Increasingly pink and
Rubricyte Normo (Erythro) 10-15 Round Dark Blue None 4:1-2:1 None
condensed larger in amount
blast

Varies polychromatic to
Meta Orthochromatic Blue- Pyknotic degeneration/
8-12 Round None 1:1- 1:2 Normachromatic - most None
Rubricyte Normoblast Purple condensed chromatin
often pale blue

Polychromatophilic Clear gray-blue


Reticulocyte 7-10 No nucleus (extruded). NA None
Erythrocyte Polychromatophilic

Biconcave disc, Pink


Red Blood Cell/
Erythrocyte 7-8 No nucleus. NA cytoplasm with central None
Discocyte
pallor

Hematology
Myelopoiesis

Hematology
Lymphopoiesis

LTSC – long term Stem cell


STSC – short term Stem cell
CMP – Common Myeloid Progenitor
CLP – Common Lymphoid Progenitor
DN – double negative

Hematology
Mnemonics
AML - “Arod, I am Lucky to live in SB” = Auer Rods, Sudan Black +, MPO +
CML - “High white old guys come to Philly for 22 year olds to sit down in their LAP” = Very high white
count, Affects old people, Low LAP score, Philadelphia Chromosome 22
ALL - “All Children use TP” = Tdt (+), PAS (+), Usually in children
CLL - “Cells Love Elderly” = Usually in elderly
tCL - “Don’t trust Hairy Men because they’re a trap” = Usually affects men, B-Cells, Trap (+)
Burkitts - “Emily B violently punched her ex in the jaw/abdomen on 8/14” = EBV, Extranodal, 8;14
translocation
“BBB” = Burkitts, B-Cells, EBV
Promyelocytic Leukemia - “PPP” = P= Pro/Plt (DIC)

Hematology
V. COAGULATION

292
BASICS!
Primary Hemostasis

Coagulation
BASICS!
Secondary Hemostasis

Coagulation
BASICS!

Secondary
Hemostasis

Coagulation
Coagulation 296
1. Which of the following is vitamin K dependent:

A. Factor XII
B. Fibrinogen
C. Antithrombin III
D. Factor VII

ANSWER: D. Factor VII

Factor VII (proconvertin) is a single-chain glycoprotein that is Vitamin K


dependent and remains stable 4-5 hours in blood. Produced in liver, it has the shortest
half life; therefore, it is the first factor affected when a vitamin K antagonist such as
warfarin is administered. BOC 5 ed
th

Coagulation 297
2. After running controls and PT was normal, PTT was abnormal. Replaced controls and got
same results. What should you do next?

A. Change out the Recombiplastin


B. Change out the CaCl
C. Rerun controls
D. Run patient tests

ANSWER: B. Change CaCl

For PTT two are reagents needed:


a. Platelet phospholipid substitute with an activator (kaolin, celite, silica, or ellagic acid
b. b. Calcium choride

For PT:
Recombiplastin or Thromboplastin source (tissue factor/TF) with calcium chloride

Coagulation 298
3. A 65-year-old patient in the emergency department has a normal D-dimer and an
elevated FDP result. These results are consistent with the presence of degradation
products of

A. Non-cross-linked fibrin
B. Cross-linked fibrin
C. Fibrinogen
D. Plasmin

ANSWER: C. Fibrinogen

The D-dimer is a specific marker of fibrinolysis. A normal D-dimer test can be


used to rule out the formation of a clot. The D-dimer test will be elevated when a clot
has formed, factor XIII has cross-linked fibrin, and the fibrinolytic system is lysing the
clot.
The FDP detection test will be abnormal whether fibrin degradation products
or fibrinogen degradation products are present. Because the D-dimer is normal in this
patient, fibrin degradation products have not formed, but fibrinogen degradation
products are present resulting in the elevated FDP result.

Coagulation 299
4. Which one of the following factors typically shows an increase in
liver disease?

A. Factor VII
B. Factor VIII
C. Factor IX
D. Factor X

ANSWER: B. Factor VIII

Liver disease affects all Vitamin-K dependent factors (II, VII, IX and X) which
will be decreased. Factor VIII is an acute phase reactant that may be elevated in liver
disease. Rodak 2007

Coagulation 300
5. Which statement regarding protein C is correct?

A. It is a vitamin K–independent zymogen


B. It is activated by fibrinogen
C. It activates cofactors V and VIII
D. Its activity is enhanced by protein S

ANSWER: D. Its activity is enhanced by protein S

Protein S functions as a cofactor of protein C and as such enhances its activity.


Activated protein C inactivates factors Va and VIIIa.

Coagulation 301
6. Coagulation factors affected by coumarin drugs are:

A. VIII, IX and X
B. I, II, V and VII
C. II, VII, IX and X
D. II, V and VII

ANSWER: C. II, VII, IX and X

Warfarin interferes with the carboxylation of vitamin K factors by interrupting the


enzymatic phase of the reaction. Factors are inhibited according to their half life, VII
having the shortest (4-5 hours) and II the longest (2-3 days). Rodak 2007

Coagulation 302
7. Which ratio of anticoagulant-to-blood is correct for coagulation
procedures?

A. 1:4
B. 1:5
C. 1:9
D. 1:10

ANSWER: C. 1:9

The optimum ratio of anticoagulant to blood is one part anticoagulant to nine parts of
blood. The anticoagulant supplied in this amount is sufficient to bind all the available
calcium, thereby preventing clotting.

Coagulation 303
8. Acute disseminated intravascular coagulation is characterized
by:

A. Hypofibrinogenemia
B. Thrombocytosis
C. Negative D-dimer
D. Shortened thrombin time

ANSWER: A. Hypofibrinogenemia

The laboratory profile for a patient with acute DIC is: increased PT, aPTT, D-
dimer and a decrease in platelets and hypofibrinogenemia.

Coagulation 304
9. Factor V Leiden promotes thrombosis by preventing:

A. Deactivation of factor Va
B. Activation of factor V
C. Activation of protein C
D. Activation of protein S

ANSWER: A. Deactivation of Factor Va

Factor V Leiden is a single-point mutation in the factor V gene that


inhibits factor Va inactivation by protein C. Activated protein C enhances
deactivation of factors Va and VIIIa

Coagulation 305
10. Patient History A 3-year-old male was admitted to a hospital with scattered
petechiae and epistaxis. The patient had normal growth and had no other
medical problems except for chickenpox 3 weeks.
Laboratory Results
Patient Reference Range
PT: 11 sec 10–13 sec
APTT: 32 sec 28–37 sec
Platelet count: 18 × 103/μL 150–450 × 103/μ

These clinical manifestations and laboratory results are consistent with which condition?

A. TTP
B. DIC
C. ITP
D. HUS

Coagulation 306
#10. Answer: C. ITP (Immune Thrombocytopenic Purpura)

These clinical manifestations and laboratory results are consistent with ITP.
ITP is an autoimmune thrombocytopenia. In children, acute ITP thrombocytopenia
occurs following a viral infection, as is the case in this 3-year-old patient. Clinical
manifestations are associated with petechiae, purpura, and mucous membrane
bleedings such as epistaxis and gingival bleeding. Abnormal laboratory tests include a
very low platelet count and a prolonged bleeding time. Other causes of
thrombocytopenia should be ruled out in patients with ITP.

Thrombotic thrombocytopenic purpura (TTP) occurs most often in adults. It is likely due to a deficiency of the
enzyme ADAMTS 13 that is responsible for breaking down large von Willebrand factor multimers. When multimers
are not broken down, clots form, causing RBC fragmentation and central nervous system impairment.

Hemolytic Uremic Syndrome occurs most often in children following a gastrointestinal infection (e.g., E. coli)

Disseminated Intravascular Coagulation is a systemic clotting initiated by activation of the coagulation cascade
due to toxins or conditions that trigger release of procoagulants (tissue factor). Multiple organ failure can occur due
to clotting

Coagulation 307
11. Abnormal Platelet Function Assay and giant platelets best
describe

A. Bernard-Soulier syndrome
B. Glanzmann thrombasthenia
C. Von Willebrand disease
D. Wiskott-Aldrich syndrome

ANSWER: A. Bernard-Soulier syndrome

Options B and C are incorrect because platelet morphology is normal even though the
bleeding time is prolonged. Option D is incorrect because this syndrome is
characterized by tiny platelets and prolonged bleeding time.

Coagulation 308
12. Which of the following enzymatically degrades the stabilized fibrin
clot?

A. Plasminogen
B. Plasmin
C. Prothrombin
D. Thrombin

ANSWER: B. Plasmin

Plasmin, the active form of plasminogen, is the enzyme responsible for degrading fibrin
into several different fragments.

Coagulation 309
13. The platelet aggregation pattern drawn below is characteristic of the
aggregating agent

A. ADP
B. Collagen
C. Ristocetin
D. Thrombin

ANSWER: B. Collagen

Collagen is the only aggregating agent that includes a single wave response preceded
by a lag phase. During the lag phase collagen stimulates platelets to release their
granule contents. Endogenous ADP released from the platelets then initiates
irreversible platelet aggregation.

Coagulation 310
Coagulation 311
14. Phlebotomist Forgetful Frank collected a tube of blood for an aPTT on John
Smith at 10:00 A.M. The blood was collected in a sodium citrate tube. At 4:30
P.M., Frank was getting ready to leave for the day when he discovered Mr.
Smith's blood specimen on his blood collection tray. So before leaving, Frank
delivered the tube of blood to the laboratory for testing. Which of the
following best describes the expected results?
A. Sodium citrate is a preservative as well as an anticoagulant, so the aPTT
result should be accurate.
B. An aPTT collected in sodium citrate will give falsely long results because
some factors are unstable in this anticoagulant.
C. A falsely long aPTT is expected because some factors deteriorate rapidly at
room temperature.
D. Exposure of the plasma to erythrocytes for several hours has probably
activated the factors, so the aPTT will be falsely short.

Coagulation 312
#14. Answer:

C. A falsely long aPTT is expected because some factors deteriorate


rapidly at room temperature.

Factors V and VIII are labile and deteriorate rapidly at room


temperature. Blood for aPTT testing should be tested within 4 hours of
draw. Sodium citrate is the appropriate anticoagulant for coagulation
procedures.

Coagulation 313
15. Patient and Family History: A 45-year-old woman visited her doctor complaining
of easy bruising and menorrhagia occurring for the past few weeks. The patient had no
history of excessive bleeding during childbirth several years earlier nor during a
tonsillectomy in childhood. Her family history was unremarkable.

Laboratory Patient Reference range


Tests
PT 45 sec 11–13 sec
APTT 125 sec 28–37 sec
Thrombin Time 14.0 sec 10–15 sec

Mixing studies (patient plasma + normal plasma): PT = 40 sec; APTT = 90 sec


Platelet count and morphology = normal
Liver function tests = normal
These clinical manifestations and laboratory results are consistent with:
A. Factor VIII inhibitor
B. Factor V inhibitor
C. Factor VIII deficiency
D. Lupus anticoagulant

Coagulation 314
#15 ANSWER: B. Factor V Inhibitor

The lack of a positive family history in this patient indicates the presence of an
acquired coagulopathy. Because both PT and APTT tests are abnormal, the clotting
factor involved is most probably in the common pathway. The lack of correction by
mixing studies suggests the presence of an inhibitor.
Factor V antibodies are the most common antibodies among the clotting
factors of the common pathway (I, II, V, and X). Factor V antibodies are reported to be
associated with surgery, some antibiotics such as streptomycin, patients who are
exposed to blood products, or the bovine form of “fibrin glue.” Patients with antibodies
to factor V may require long-term therapy with immunosuppressive drugs. Acute
bleeding episodes may be treated by platelet transfusions. The PT test is normal in
patients with factor VIII deficiency and factor VIII inhibitor. Lupus anticoagulant is not
present with bleeding unless associated with coexisting thrombocytopenia.

Coagulation 315
RIDDLE TIME!

What kind of chemical element


hates to be a follower?

Lead

316
317
VI. IMMUNOLOGY

318
1. Antinuclear antibody tests are performed to help diagnose:

A. Acute Leukemia
B. Lupus erythematosus
C. Hemolytic anemia
D. Crohn disease

ANSWER: B. Lupus erythematosus

ANA detects circulating antibodies to nuclear antigens in systemic rheumatic diseases.

Immunology 319
2. In the anti-double-stranded DNA procedure, the antigen most
commonly utilized is:

A. Rat stomach tissue


B. Mouse kidney tissue
C. Crithidia lucillae
D. Toxoplasma gondii

ANSWER: C. Crithidia lucillae

The Crithidia substrate has giant mitochondrion containing native DNA that is free from
contaminating histone antigens.

Immunology 320
Immunology 321
3. In the indirect fluorescent ANA, the pattern shown in the picture
indicates the presence of antibody to?

A. RNP
B. Sm
C. RNA
D. DNA

ANSWER: D. DNA

The picture shows a rim pattern for ANA. This can be caused by antibodies to double
and single stranded DNA seen in SLE in high titers and in lower titers in other
rheumatic diseases.

Immunology 322
The first stage of mitosis is called prophase. During this
phase the chromatin condenses. The nuclear membrane
disappears as the cell moves into metaphase.

At metaphase the chromosomes are lined up in the


center of the cell.

Immunology 323
In anaphase the chromatids (the chromosomal arms)
separate and are pulled toward the poles.

In telophase, the chromosomes reach the poles. The


nuclear membrane starts to reassemble. However, the cell
has not divided.

DAUGHTER CELLS

Immunology 324
Homogeneous pattern

Immunology
Speckled pattern

Immunology
Centromere pattern

Immunology
Nucleolar pattern

Immunology
PCNA pattern

Immunology
Which pattern is this?

Centromere

Clinical significance:
(CREST)
Calcinosis cutis
Raynaud’s phenomenon
Esophageal dysfunction
Sclerodactyly
Telangiectasia

Immunology
Which pattern is this?

Negative

Notice there is no clearly discernible


pattern in the interphase cells

Immunology
Immunology
4. Rheumatoid factors are defined as:

A. Antigens found in the sera of patients with rheumatoid arthritis


B. Identical to the rheumatoid arthritis precipitin
C. Autoantibodies with specificity for the Fc portion of the IgG molecule
D. Capable of forming circulating immune complexes only when IgM-type
autoantibody is present

ANSWER: C. Autoantibodies with specificity for the Fc portion of the IgG molecule

Rheumatoid factor is an autoantibody to the Fc portion of the immunoglobulin


molecule.

Immunology 333
5. An acute phase protein that binds to the membrane of certain
microorganisms and activates the complement system?

A. C-reactive protein
B. Tumor necrosis factor Alpha
C. Neutrophils
D. Kinins

ANSWER: A. C-reactive protein

Kinins act on smooth muscle; neutrophils are phagocytic cells; and TNF are released
by activated macrophages.

Immunology 334
6. Which of the below figures demonstrates a reaction pattern of
identity?

A. Figure #1
B. Figure #2
C. Figure #3
D. Figure #4

ANSWER: A. Figure #1
Continuous joining precipitin lines form when the 2 antigens are identical.

Figure #2 is Non-Identity
Figure #3 is 2 different antigenic molecular species
Figure #4 is Nonspecific precipitin reaction is demonstrated

Immunology 335
7. The presence of HbsAg, anti-HBc and often HbeAg is
characteristic of:

A. Early acute phase HBV hepatitis


B. Early convalescent phase HBV hepatitis
C. Recovery phase of acute HBV hepatitis
D. Past HBV infection

ANSWER: A. Early acute phase HBV hepatitis

The markers listed appear early during hepatitis B infection; HbsAg and HBeAg
disappear prior to convalescence and recovery.

Immunology 336
8. The disappearance of HBsAg and HBeAg, the persistence of
anti-HBc, the appearance of anti-HBs, and often of anti-Hbe
indicate:

A. Early acute phase HBV hepatitis


B. Early convalescent phase HBV hepatitis
C. Recovery phase of acute HBV hepatitis
D. Carrier state of acute HBV infection

ANSWER: C. Recovery phase of acute HBV hepatitis

Anti-HBs and anti-Hbe are associated with recovery and development of immunity in
hepatitis B, while HBsAg and HBeAg are antigens from HBV that are present during
infectious stage.

Immunology 337
Immunology 338
Note: HBcAb positive during window period (also a recall)

Immunology 339
9. Which of the following immunoglobulins is present in the
highest concentration in normal human serum?

A. IgM
B. IgG
C. IgA
D. IgE

ANSWER: B. IgG

IgG has the highest concentration in normal sera. Approximately 75% in the blood.

Immunology 340
10. The immunoglobulin class typically found to be present in
saliva, tears and other secretions is:

A. IgM
B. IgG
C. IgA
D. IgE

ANSWER: C. IgA

A secretory IgA molecule is composed of 2 units of 2 heavy chains and 2 light chains.
These chains are joined by the J chain, and are protected from the harsher
environment where there are secreted by an additional chain called secretory piece.
Stevens 2003

Immunology 341
11. Which immunoglobulin appears first in the primary immune
response?

A. IgM
B. IgG
C. IgA
D. IgE

ANSWER: A. IgM

The first antibody to appear in the primary immune response to an antigen is IgM. The
titer of antiviral IgM (e.g., IgM antibody to cytomegalovirus [anti-CMV]) is more specific
for acute or active viral infection than IgG and may be measured to help differentiate
active from prior infection.

Immunology 342
Immunology 343
12. What substance is detected by the rapid plasma reagin (RPR) and
Venereal Disease Research Laboratory (VDRL) tests for syphilis?

A. Cardiolipin
B. Anticardiolipin antibody
C. Anti-T. pallidum antibody
D. Treponema pallidum

ANSWER: B. Anticardiolipin antibody

Reagin is the name for a nontreponemal antibody that appears in the serum of
syphilis-infected persons and is detected by the RPR and VDRL assays. Reagin reacts
with cardiolipin, a lipid-rich extract of beef heart and other animal tissues.

Immunology 344
Treponemal tests

1. Darkfield microscopy
2. Fluorescent treponemal antibody absorption test (FTA-Abs)
3. Treponema pallidum Immobilization test (TPI)
4. Microhemagglutination Assay for T. pallidum

NON-Treponemal tests

1. Venereal Disease Research Laboratory (VDRL)


2. Rapid Plasma Reagin Test (RPR)

Immunology 345
13. The plate below is a Wright’s-stained peripheral blood film, 1,000×. What
is the most appropriate classification of the white blood cells (WBCs) present
in this field?
A. Reactive (atypical) lymphocytes
B. Large lymphoblasts exhibiting L2 morphology
C. The M4 subtype of acute granulocytic leukemia
D. Monocytes

Immunology 346
#13. ANSWER A. Reactive Lymphocytes

These cells are lymphocytes characteristic of those found in viral infections such
as infectious mononucleosis. In these conditions, the WBC count is increased (usually
15–25 × 103/μL), and lymphocytes account for the majority of the WBCs. Reactive
lymphocytes are larger than normal. The cytoplasm is increased in volume and may be
vacuolated, and the edges of the cell are often scalloped and basophilic. The nuclear
chromatin pattern is open and reticular.

Immunology 347
14. Which of the following is used to detect allergen specific IgE?

A. RIST
B. EIA
C. RAST
D. IEP

ANSWER: C. RAST

RAST tests for allergen specific IgE. Stevens 2003

Immunology 348
Radioimmunoassays

1. The radioallergosorbent test (RAST) is used to detect IgE against specific allergens. The test can be
performed as a competitive or noncompetitive assay.
2. The radioimmunosorbent Test RIST measures total IgE

EIA: Enzyme Immunoassay (ANA, FTA-Abs, HIV, Hepatitis)


IEP: Immunoelectrophoresis (commonly used to determine heavy and light chains involved like in
monoclonal gammopathies in serum or bence jones protein in urine)

Immunology 349
15. SITUATION: A differential shows reactive lymphocytes, and the
physician suspects a viral infection is the cause. What is the expected
laboratory finding in a patient with a cytomegalovirus (CMV) infection?

A. Heterophile antibody: positive


B. Epstein–Barr virus (EBV)–immunoglobulin (IgM): positive
C. Direct antiglobulin test (DAT): positive
D. CMV–IgM: positive

ANSWER: D. CMV–IgM: positive

If both the heterophile antibody test and the EBV-IgM tests are negative in a patient
with reactive lymphocytosis and a suspected viral infection, the serum should be
analyzed for IgM antibodies to CMV. CMV belongs to the herpes virus family and is
endemic worldwide. CMV infection is the most common cause of heterophile-negative
infectious mononucleosis

Immunology 350
16. What are the steps of PCR?

A. Denaturation, Annealing and Transcription


B. Annealing, Denaturation and Transcription
C. Transcription, Denaturation and Annealing
D. Denaturation, Transcription and Annealing

ANSWER: A. Denaturation, Annealing and Transcription

Immunology 351
PCR Technique

Immunology
353
VII. MICROBIOLOGY

354
1. The urine antigen test is the most common laboratory assay used for
the diagnosis of legionellosis?

A. Eikenella
B. Legionella
C. Pasteurella
D. Actinobacillus

ANSWER: B. Legionella

Legionella spp. are aquatic organisms that may be found in various water systems, including
humidifiers, whirlpools, and air conditioning chillers. They are resistant to commonly used concentrations of
chlorine.

Microbiology 355
2. This member of the Enterobacteriaceae is shown on triple sugar iron agar,
lysine iron agar, urea agar, citrate agar , phenylanine deaminase agar, and
motility indole ornithine agar. It should be identified as:

The correct answer is highlighted below

A. Salmonella spp.
B. Shigella spp.
C. Proteus vulgaris
D. Citrobacter freundii

ANSWER: A. Salmonella spp.

Salmonella spp. is the correct answer because the isolate is a non-lactose and
sucrose fermenter on TSIA, positive for H2S, lysine decarboxylase positive, urea
negative, citrate positive, phenylalanine deamination negative, indole negative, and
ornithine positive

Microbiology 356
3. A curved gram-negative rod producing oxidase-positive colonies on blood
agar was recovered from a stool culture. Given the following results, what is
the most likely identification?

A. Vibrio cholerae
B. Vibrio parahaemolyticus
C. Shigella spp.
D. Salmonella spp

ANSWER: B. Vibrio parahaemolyticus

V. parahaemolyticus appear as green colonies on TCBS agar, whereas V. cholerae


appear as yellow colonies on TCBS.
V. cholerae is the only Vibrio species that causes a positive string test.

Microbiology 357
4. The HACEK group of organisms (Haemophilus aphrophilus, Actinobacillus
actinomycetemcomitans, Cardiobacterium hominis, Eikenella corrodens, and
Kingella spp.) are all known for which type of infection?

A. Urinary tract
B. Endocarditis
C. Pharyngitis
D. Tonsilitis

ANSWER: B. Endocarditis

Blood cultures growing small gram-negative rods shoud alert the


microbiologist to the possiblility of infection with one of the five HACEK
organisms. Although responsible for less than 5% of bacterial endocarditis
overall, greater than half of endocarditis cases caused by gram-negative rods
result from one of them.

Microbiology 358
5. Illustrated in this photograph are colonies growing on the surface of a
chocolate agar plate. The colonies were recovered from a subcutaneous infection
of the forearm and had a bleach odor. The most likely identification is:

Choose the single best answer

a. Moraxella lacunata
b. Haemophilus aphrophilus
c. Eikenella corrodens
d. Capnocytophaga canimorsus

ANSWER: C. Eikenella corrodens

Eikenella corrodens produces a bleachlike odor. Ciulla 4th edition p636

Microbiology 359
6. Illustrated in this photograph are two Middlebrook 7H-11 plates incubated at
35°C, on which are growing colonies of an unknown Mycobacterium species. The
plate on the left was incubated in the dark; the plate on the right had been
exposed to light for 24 hours. The most likely identification is:

The correct answer is

A. Mycobacterium gordonae
B. Mycobacterium kansasii
C. Mycobacterium tuberculosis
D. Mycobacterium marinum

ANSWER: B. Mycobacterium kansasii

Of the Mycobacterium species listed in this exercise, M. kansasii and M. marinum are the
two photochromogens, fitting the description of the colonies included in the question. Of these,
M. marinum would not grow at 35°C; therefore, the intended answer in this exercise is M.
kansasii.

Microbiology 360
7. Illustrated in this photograph, reading from left to right,
are biochemical reactions for indole, Voges Proskauer,
Simmons citrate and Christensen's urea. The bacterial
species providing the best fit for the reactions illustrated is:

A. Salmonella typhimurium
B. Citrobacter koseri
C. Klebsiella oxytoca
D. Enterobacter cloacae

ANSWER: C. Klebsiella oxytoca

The bacterial species that best fits the four positive reactions
of the characteristics shown here is Klebsiella oxytoca. The partial or
weak urease reaction (slant only) in particular is in keeping with the
genus Klebsiella, although some Enterobacter species may show a
similar reaction.

Microbiology 361
8. The reactions seen in the tubes presented in this photograph reveals a
motile, gas producing, lactose fermenter that produces acetyl methyl carbinol
in VP broth and utilizes sodium citrate as the sole source of carbon. The most
likely identification is:

A. Citrobacter freundii
B. Enterobacter aerogenes
C. Klebsiella pneumoniae
D. Proteus mirabilis

ANSWER: B. Enterobacter aerogenes

Enterobacter aerogenes ferments lactose, is motile, does not


produce indole and is both VP and citrate positive, consistent with the
reactions seen here

Microbiology 362
9. Illustrated in this photograph is an H & E-stained section of a gastric biopsy
obtained from a patient with suspected peptic ulcer disease. Based on the
biopsy findings, the bacterial species that would most likely be recovered is:

A. Salmonella typhi
B. Campylobacter hyointestinalis
C. Yersinia paratuberculosis
D. Helicobacter pylori

ANSWER: D. Helicobacter pylori

Helicobacter pylori is a major cause of peptic ulcer disease. The


microorganism can be isolated from gastric biopsy on SBA, Brucella, and Skirrow's
agars incubated microaerophilically.

Microbiology 363
10. Which of the following media contains the X and V factors necessary for
the growth of Haemophilus influenzae:

A. MacConkeys agar
B. Sheep blood agar
C. Chocolate agar
D. Brain-heart infusion agar

ANSWER: C. Chocolate agar

Chocolate agar contains both the X factor (hemin) and V factor


(coenzyme nicotine adenine dinucleotide) which are necessary for growth of H.
influenzae. It is prepared by adding sheep blood to agar at 80o C to lyse the
red cells.

Microbiology 364
11. The Quellung test is useful for which of the following :

A. Differentiate between Staphylococcus and Streptococcus


B. Serological typing of Streptococcus pneumoniae
C. Isolation of various strains of Staphylococcus
D. Isolation of various strains of Streptococcus

ANSWER: B. Serological typing of Streptoccocus pneumoniae

A precipitin reaction seen microscopically with methylene blue stain


(microprecipitin reaction) occurs between the carbohydrate of the capsule of S.
pneumoniae and anticapsular antibody. The antibody may be type specific or
polyvalent. Binding of antibodies to the bacteria causes the capsule to swell, identifying
the organisms as S. pneumoniae.

Microbiology 365
12. A gram-positive coccus that is catalase positive, nonmotile, lysostaphin
resistant, and modified oxidase positive is best identified as a member of the
genus

A. Micrococcus
B. Lactococcus
C. Pediococcus
D. Staphylococcus

ANSWER: A. Micrococcus

Staphylococci and micrococci are both catalase positive gram-positive cocci.


Staphylococci are more clinically significant, so it is important to differentiate Micrococcus from
Staphylococcus. Micrococci are modified oxidase positive, whereas Staphylococci are
negative.

Microbiology 366
13. A negative PYR (L-pyrolidonyl-anaphthylamide) test is demonstrated
by:

A. Enterococcus faecalis
B. Enterococcus faecium
C. Streptococcus pyogenes
D. Viridans streptococci

ANSWER: D. Viridans streptococci

Viridans streptococci do not produce the enzyme pyroglutamyl


aminopeptidase and, therefore, in the PYR test do NOT produce a
positive or red color. The PYR test is used predominantly for the
presumptive identification of group A streptococci and Enterococcus.

Microbiology 367
14. Enterococcus faecium is characteristically

A. Inhibited by the presence of bile in culture media


B. Able to grow in the presence of high concentrations of salt
C. PYR negative
D. Beta-hemolytic

ANSWER: B. Able to grow in the presence of high concentrations of salt

Enterococcus faecium is an important agent of human infection. Their


differentiation from other enterococcal strains is of importance because of their
resistance to most clinically useful antimicrobial agents, including vancomycin. The
ability to tolerate a high concentration of salt is characteristic of the clinically significant
species of Enterococcus. E. faecium is PYR positive and is usually nonhemolytic.

Microbiology 368
15. The rapid latex agglutination assay for S. aureus detects what?

A. Clumping factor and protein A


B. Protein A
C. Protein C
D. Clumping factor and M protein

ANSWER: A. Clumping factor and protein A

Latex agglutination assay detects clumping factor and protein A on the surface
of S. aureus. Ciulla 4 edition
th

Microbiology 369
16. A yellow colony from a wound culture tested catalase positive, coagulase negative.
The organism stained as gram positive cocci in clusters. Which of the following tests
would differentiate between a coagulase negative Staphylococcus and Micrococcus?

A. Novobiocin susceptibility
B. Leucine aminopeptidase production
C. Furazolidone (100 ug/disk) susceptibility
D. Bile esculin

ANSWER: C. Furazolidone (100 ug/disk) susceptibility

Microccocus and Staphylococcus can be differentiated by susceptibility to


furazolidone (100ug/disk). Staphylococcus is susceptible and Micrococcus is resistant.

Microbiology 370
17. The etiologic agent of the disease erysipelas is

A. Staphyloccus aureus
B. Streptobacillus moniliformis
C. Streptococcus agalactiae
D. Streptococcus pyogenes

ANSWER: D. Streptococcus pyogenes

Erysipelas results from person-to-person transmission of group A streptococci.


Symptoms occur when nasopharyngeal infection spreads to the face. The rare complication of
an upper respiratory infection with Streptococcus pyogenes is characterized by sensations of
burning and tightness at the site of invasion. Erythema associated with this superficial cellulitis
rapidly spreads with an advancing elevated margin.

Microbiology 371
18. This parasite is found in blood.

A. Plasmodium schizont
B. Leishmania amastigote
C. Plasmodium ring form
D. Trypanosoma promastigote

ANSWER: C. Plasmodium ring form

This organism is a Plasmodium ring form. Note that this form consists of a
"ring" of cytoplasm that is connected by a chromatin dot. Plasmodium species are the
only parasites listed that actually invade unsuspecting red blood cells.

Microbiology 372
19. A human spirochetal disease associated with contact with rats is:

A. Lyme disease
B. Relapsing fever
C. Leptospirosis
D. Syphilis

ANSWER: C. Leptospirosis

Human infections of leptospirosis have classically been associated with


contact with rats, particularly soil, water and materials soiled with rat urine, other
domestic animals may be infected as well.

Microbiology 373
20. Illustrated in this photograph is a blood agar plate inoculated with a beta
hemolytic streptococcus obtained from a throat culture of a patient with acute
pharyngitis. A 0.04ug bacitracin (left) disk and a SXT disk (right) had been
placed in the areas of streaking. The reactions observed allow the most likely
presumptive identification of a Streptococcus belonging to Lancefield group:

A. Group A
B. Group B
C. Group C
D. Group G

ANSWER: A. Group A
The zone of growth inhibition around the bacitracin (left) disk is
presumptive evidence for a beta hemolytic streptococcus, group A.

Microbiology 374
21. A 30 year old man with chronic persistent hepatitis developed an
abscess in the right hepatic lobe. A needle aspiration revealed purulent
material. The small colonies shown in the upper photograph were
isolated after 36 hours incubation at 35°C. They emitted a butter scotch
odor. The lower photomicrograph reveals the gram stain features. The
antigen most likely carried by this isolate is:

A. Group A
B. Group B
C. Group G
D. Group F

ANSWER: D. Group F
The tiny colonies seen in the upper photograph are surrounded
by a narrow zone of beta hemolysis. The lower photomicrograph
confirms that the isolate is a streptococcus. The small size of the
colonies and the butter scotch odor are clues to the identification of
Streptococcus anginosus (S. milleri in the European literature), which
characteristically carries the F antigen.

Microbiology 375
22. Exudate from a burn submitted on a swab for bacterial culture grew a gram-negative
bacillus that was oxidase-positive, produced an alkaline slant and butt on TSIA, was resistant
to numerous antibiotics, and produced a green, water-soluble pigment. This organism would
be identified as:

A. Pseudomonas fluorescens
B. Pseudomonas aeruginosa
C. Stenotrophomonas maltophilia
D. Burkholderia cepacia

ANSWER: B. Pseudomonas aeruginosa

Pseudomonas aeruginosa is the correct answer. Pseudomonas fluorescens,


Stenotrophomonas maltophilia, and Burkholderia cepacia do not produce a green, water-
soluble pigment.

Microbiology 376
23. Which of the following would be the most appropriate temperature for long
term storage of viral cultures:

A. 4° C
B. -20° C
C. -70° C
D. Room temperature

ANSWER: C. -70C

Cultures can be stored at either -20° or -70°, but -70° is preferred for long term
storage.

Chemistry 377
24. Which of the following tests is most appropriate for the presumptive
identification of Clostridium perfringens?

A. SPS sensitivity test


B. Reverse CAMP test
C. Cytotoxin assay
D. Esculin hydrolysis

ANSWER: B. Reverse CAMP test


A reverse CAMP test aids in the identification of Clostridium perfringens. In this test, a
single straight streak of Streptococcus agalactiae is made down the center of the plate.
Suspected C. perfringens isolates are inoculated at right angles to the S. agalactiae inoculum.
After anaerobic incubation, C. perfringens will exhibit enhanced hemolysis at the intersection
where the two species meet.

Microbiology 378
25. A gram-positive branching filamentous organism recovered from a sputum
specimen was found to be positive with a modified acid-fast stain method.

A. Bacillus spp.
B. Nocardia spp.
C. Corynebacterium spp.
D. Listeria spp.

ANSWER: B. Nocardia spp.

Nocardia spp. should be suspected if colonies that are partially acid fast by the
traditional method are positive with the modified acid-fast method using Kinyoun stain and
1% sulfuric acid as the decolorizing agent. The other organisms listed are negative for acid-fast
stain. Nocardia is an opportunistic pathogen, and cultures typically have a musty basement
odor.

Microbiology 379
26. Bacillus cereus has been implicated as the etiologic agent in cases of

A. Food poisoning
B. Impetigo
C. Pelvic inflammatory disease
D. Toxic shock syndrome

ANSWER: A. Food poisoning

Bacillus spp. are gram-positive, spore-forming bacilli widely found in


the environment. Bacillus cereus is of particular interest as an etiologic agent
of human cases of food poisoning. This enterotoxin-producing microorganism
is most commonly associated with cases of food poisoning following ingestion
of reheated rice served at Asian restaurants.

Microbiology 380
27. A gram-positive spore-forming bacillus growing on sheep-blood agar
anaerobically produces a double zone of β-hemolysis and is positive for
lecithinase. What is the presumptive identification?

A. Bacteroides ureolyticus
B. Bacteroides fragilis
C. Clostridium perfringens
D. Clostridium difficile

ANSWER: C. C. perfringens

C. perfringens produces a double zone of β-hemolysis on blood agar, which makes


identification relatively easy. The inner zone of complete hemolysis is caused by a θ-toxin and
the outer zone of incomplete hemolysis is caused by an α-toxin (lecithinase activity). The
Bacteroides spp. are gram-negative bacilli, and C. difficile is lecithinase negative and does not
produce a double zone of β-hemolysis.

Microbiology 381
Clostridium perfringens
Double zone of hemolysis and beta lactamase
Microbiology 382
28. Propionibacterium acnes is most often associated with:

A. Normal oral flora


B. Post-antibiotic diarrhea
C. Tooth decay
D. Blood culture contamination

ANSWER: D. Blood culture contamination

Propionibacterium acnes is part of the normal flora of the skin, so it is


frequently isolated from improperly collected blood cultures.

Microbiology 383
29. A fastidious gram-negative bacillus was isolated from a case of periodontal
disease, which upon darkfield examination was noted to have gliding motility.
The most likely identification of this etiologic agent would be:

A. Capnocytophaga
B. Chromobacterium
C. Kingella
D. Plesiomonas

ANSWER: A. Capnocytophaga

Capnocytophaga spp. are fermentative gram-negative bacteria that inhabit the oral
cavity of humans. These organisms have been identified as a cause of disease in the oral cavity,
and in compromised hosts they have been implicated in systemic disease isolated from
cerebrospinal fluid, pleural fluid, and pulmonary secretions. The gliding motility is best
observed during the log phase of growth and can be demonstrated by darkfield microscopy and
on sheep blood agar by the production of concentrically spreading growth around primary
colonies.

Microbiology 384
30. A leg-wound culture from a hospitalized 70-year-old diabetic man grew
motile, lactose-negative colonies on MacConkey agar. Given the following
biochemical reactions at 24 hours, what is the most probable organism?

A. Proteus vulgaris
B. Serratia marcescens
C. Proteus mirabilis
D. Enterobacter cloacae

ANSWER: B. Serratia maercescens


S. marcescens has been implicated in numerous nosocomial infections
and is recognized as an important pathogen with invasive properties. Gelatin
hydrolysis and DNase are positive for both the Proteus spp. and Serratia, but
the negative urease and phenylalanine deaminase are differential. E.
cloacae does not produce DNase, gelatinase, or lysine decarboxylase.

Microbiology 385
31. On Gram stain, a morphology that resembles "seagull wings" is most
characteristic of

A. Campylobacter jejuni
B. Neisseria gonorrhoeae
C. Plesiomonas shigelloides
D. Yersinia pseudotuberculosis

ANSWER: A. Campylobacter jejuni

Fresh isolates of Campylobacter jejuni on Gram stain characteristically


reveal a "gull-wing" appearance. These gram-negative bacilli are motile with a
typical darting pattern on wet mounts. They stain poorly using the Gram stain
method, and it is recommended that carbolfuchsin or basic fuchsin be
substituted for the counterstain safranin.

Microbiology 386
32. Four blood cultures were taken over a 24-hour period from a 20-year-old
woman with severe diarrhea. The cultures grew motile (room temperature),
gram-negative rods. A urine specimen obtained by catheterization also showed
gram-negative rods, 100,000 col/mL. Given the following results, which is the
most likely organism?

A. Proteus vulgaris
B. Salmonella typhi
C. Yersinia enterocolitica
D. E. coli

ANSWER: D. E. coli

Typically, the IMViC reactions for the organisms listed are:


E. coli (++00) S. typhi (0+00) Y. enterocolitica (V+00) P. vulgaris (++00)

Microbiology 387
Microbiology 388
IMViC

++-- --++ -+-+ -+-- -+++ +-++ ++-+


E. coli Klebsiella pneumoniae Salmonella Shigella Proteus mirabilis Klebsiella oxytoca Citrobacter Koseria

Edwardsiella Enterobacter Citrobacter freundii Providencia

Proteus Vulgaris Serratia

Morganella

Yersinia

Microbiology 389
KIA
A/A GAS A/A K/A GAS K/A

E. coli Edwardsiella Shigella


Citrobacter freundii Salmonella Providencia
Klebsiella Citrobacter Freundii Yersinia
Enterobacter Citrobacter Koseri
Serratia
Proteus
Morganella

Microbiology 390
33. Semisolid transport media such as Amies, Stuart, or Cary–Blair are
suitable for the transport of swabs for culture of most pathogens except:

A. Neisseria gonorrhoeae
B. Enterobacteriaceae
C. Campylobacter fetus
D. Streptococcus pneumoniae

ANSWER: A. N. gonorrhoeae

Specimens for culture of N. gonorrhoeae are best if plated immediately


or transported in a medium containing activated charcoal to absorb inhibitory
substances that hinder their recovery.

Microbiology 391
34. The best specimen for recovery of the mycobacteria from a sputum
sample is:

A. First morning specimen


B. 10-hour evening specimen
C. 12-hour pooled specimen
D. 24-hour pooled specimen

ANSWER: A. First morning specimen

Contamination by fungi and other bacteria contributes to lower yields of


mycobacteria in a 24-hour sample. The first morning specimen collected by
expectoration or nebulization produces the highest concentration of mycobacteria in
sputum.

Microbiology 392
35. How long should Mycobacterium tuberculosis–positive cultures be kept by the
laboratory after identification and antibiotic susceptibility testing have been performed?

A. 1–2 months
B. 2–4 months
C. 5–6 months
D. 6–12 months

ANSWER: D. 6-12 months

Standard therapy using INH and rifampin for classic, uncomplicated pulmonary
tuberculosis is 9 months. The patient may not respond to therapy, even when the
organism is susceptible to the antibiotics in vitro; therefore, cultures must be kept for
up to 1 year in order to facilitate testing of additional antibiotics should the infection
become refractory to therapy.

Microbiology 393
HELPFUL MNEMONICS!
Non-Motile Enterics –
“Kill Yourself” = Klebsiella, Yersinia, & Shigella
H2S on TSI –
“Ed gets Protein from Blackened Citrus Salmon” = Edwardsiella, Proteus, Citrobacter, &
Salmonella
Urease Positive –
“Urea is PP, Mmkay?” = Prodeus, Providencia, Morganella, Klebsiella, & Yersinia
PDA –
“PDA MPP” = Morganella, Proteus, & Providencia
Citrate Positive Salmonella –
“S. enterica” = C for Citrate. (S. enterica is -+-+ & S. typhi is -+--)
Indole Negative Klebsiella –
“No Mo Indole” = (K. pheumo is --++ & K. oxytoca is +-++)

Microbiology 394
HELPFUL MNEMONICS!

Indole Positive Citrobacter –


“I for Indole” = (C. koseri is ++-+ & C. f/b are -+-+)
Lysine Decarbox Positive Citrobacter –
“CA is a Liar” = C. aerogenes is Lysine Decarbox +
Argenine Decarbox Positive Citrobacter –
“CC Argues” = C. cloacae is Arg Decarbox +
Lactose Fermenters –
“LA Cronies Enter the Eco Klub” = Cronobacter, Enterobacter, E. coli, & Klebsiella
IMVC –
“MEEP” = Morganella, E. coli, Edwardsiella, Proteus vulgaris are all ++--
“Enter Sir, Keep Having Croissants” = Enterobacter, Serratia, K. pneumo, Hafnia, & Cronobacter are all --++

Microbiology 395
396
VII. MYCOLOGY

397
1. Identify the fungi illustrated on the photo below?

A. Alternaria
B. Aspergillus
C. Fusarium
D. Penicillium

ANSWER: A. Alternaria

Alternaria alternata looks like tangled Christmas lights but with grenades.

Mycology 398
Alternaria

Mycology 399
2. A neonatal blood culture collected through a catheter grows a small
yeast as shown below. Growth is enhanced around olive oil saturated
discs. The organism isolated is?

A. Candida tropicalis
B. Malassezia furfur
C. Candida lipolytica
D. Malassezia pachydermatis

ANSWER: B. Malassezia furfur

Malassezia furfur causes catheter-related sepsis, requires lipids for growth and
is a small yeast with a wide bond. It appears to look like spaghetti with meatballs and is
an olive oil loving fungi.

Mycology 400
3. Examination of a fungal culture from a bronchial washing reveals
white, cottony aerial mycelium. A tease preparation in lactophenol cotton
blue shows the structures in the image:
The most rapid test for definitive identification is:

A. Nucleic acid probe


B. Animal inoculation
C. Exoantigen test
D. Slide culture

ANSWER: A. Nucleic acid probe

Nucleic acid probe test specific for Coccidioides immitis can be


completes in <4 hours

Mycology 401
4. A 44-year-old gardener pricked herself with a rose thorn. A subcutaneous fungal
infection characterized by the development of necrotic ulcers followed this direct
inoculation of fungal spores into the skin. The causative fungus was cultured as a
small yeast form at 35°C and as a mould at room temperature with delicate hyphae
and conidia. This disease is:

A. Blastomycosis
B. Chromomycosis
C. Mycetoma
D. Sporotrichosis

ANSWER: D. Sporotrichosis

Sporothrix schenckii is the agent of sporotrichosis. It usually enters the skin by


traumatic implantation. This fungus grows in vitro as small yeasts at 35°C and as a
mould at room temperature (22-30°C) with delicate hyphae and conidia.

Mycology 402
5. Broad, coenocytic hyphae found in tissue would be most typical of infection
with

A. Aspergillus
B. Blastomyces
C. Microsporum
D. Rhizopus

ANSWER: D. Rhizopus
Rhizopus and other fungal agents of mucormycosis are characterized
by having coenocytic (nonseptate) hyphae. The finding of broad, nonseptate
hyphal elements in sterile body fluids or tissue can provide rapid confirmation
of a clinical diagnosis of mucormycosis. The other moulds listed have septate
hyphae.

Mycology 403
6. Which of the following yeasts is characteristically positive for germ tube production?

A. Candida tropicalis
B. Candida kefyr (pseudotropicalis)
C. Cryptococcus neoformans
D. Candida albicans

ANSWER: D. Candida albicans

C. albicans and Candida dubliniensis, a variant of C. albicans, are the only


yeasts that produce germ tubes within 1–3 hours of incubation at 37°C. C. tropicalis
produces pseudohyphae after incubation for 3 hours, which may be mistaken for germ
tubes. A careful evaluation of the tube origin for constriction is required to avoid a false-
positive interpretation

Mycology 404
7. For which clinical specimens is the KOH direct mount technique for
examination of fungal elements used?

A. Skin
B. CSF
C. Blood
D. Bone marrow

ANSWER: A. Skin

A solution of 10% KOH is used for contaminated specimens such


as skin, nail scrapings, hair, and sputum to clear away background
debris that may resemble fungal elements. Normally sterile specimens
(CSF, blood, and bone marrow) do not require KOH for clearing.

Mycology 405
8. The India ink stain is used as a presumptive test for the presence of which
organism?

A. Aspergillus niger in blood


B. Cryptococcus neoformans in CSF
C. Histoplasma capsulatum in CSF
D. Candida albicans in blood or body fluids

ANSWER: B. Cryptococcus neoformans in CSF

Meningitis caused by C. neoformans is diagnosed through culture, biochemical


reactions, and rapid agglutination tests for cryptococcal antigen. The India ink test is not
diagnostic for cryptococcal meningitis because positive staining results are demonstrated in
less than 50% of confirmed cases. A positive India ink test shows yeast cells in CSF with a
surrounding clear area (the capsule) because the capsule of C. neoformans is not penetrated
by ink particles.

Mycology 406
9. Which Aspergillus species, recovered from sputum or bronchial mucus, is
the most common cause of pulmonary aspergillosis?

A. A. niger
B. A. flavus
C. A. fumigatus
D. All of these options

ANSWER: C. A. fumigatus

A. fumigatus is most often associated with compost piles and is found


in the soil of potted plants. A. niger is the cause of cavitary fungus ball lesions
of the lungs and nasal passages.

Mycology 407
10. Which dematiaceous mold forms flask-shaped phialides, each with a
flask-shaped collarette?

A. Phialophora spp.
B. Exophiala spp.
C. Wangiella spp.
D. All of these options

ANSWER: A. Phialophora spp.

Phialophora, Exophiala, and Wangiella all produce phialides, but the last two
genera form elongated, tubelike phialides without a collarette, as opposed to the flask-
shaped phialides of Phialophora, which contain clusters of conidia at the tips
Mycology 408
409
IX. PARASITOLOGY

410
1. The term “internal autoinfection” is generally used in referring to
infections with?

A. Ascaris lumbricoides
B. Necator americanus
C. Trichuris trichiura
D. Strongyloides stercoralis

ANSWER: D. Strongyloides stercoralis

Strongyloides stercoralis rhabditiform larvae are capable of transforming into filariform


(infective) larvae in the intestines of immunocompromised patients. This establishes an
autoinfective cycle. (Murray 2007)

Parasitology 411
2. The best method to demonstrate the ova shown in the picture is?

A. Acid-ether concentration
B. Cellophane tape preparation
C. Formalin-ether concentration
D. Zinc sulfate flotation

Answer: B. Cellophane tape preparation

The ova of E. vermicularis cannot be demonstrated in a routine ova and parasite examination.
The adult female Enterobius worm migrates out of the anus, and lays her eggs in the perianal
folds. A scotch tape preparation of the skin of the perianal folds is used to collect ova. (BOC 5th
Edition)

Parasitology 412
3. An organism was isolated from an eye wash of a patient with a
keratitis (cornea infection) who had been wearing contact lenses
for the past 2 years. What is the name of the causative agent?

A. Naegleria spp.
B. Acanthamoeba spp.
C. Entamoeba histolytica
D. Trichomonas vaginalis

ANSWER: B. Acanthamoeba spp.

Acanthamoeba is a large trophozoite with spiculated cytoplasm characteristic. Eye


infections caused by this organism have been documented in contact lens wearers
who do not properly disinfect lenses. (Harr)

Parasitology 413
4. Primary Amoebic Encephalitis may be caused by:?

A. Entamoeba coli
B. Dientamoeba fragilis
C. Endolimax nana
D. Naegleria fowleri

ANSWER: D. Naegleria fowleri

Naegleria fowleri is the etiologic agent of primary amoebic encephalitis. (Murray 2007)

Parasitology 414
5. Massive hemolysis, blackwater fever, and central nervous system
involvement are most common with:

A. Plasmodium vivax
B. Plasmodium falciparum
C. Plasmodium ovale
D. Plasmodium malariae

ANSWER: B. Plasmodium falciparum

The pathogenic sequelae of malarial infections with P. falciparum are the most severe
of the five species of human malaria. They can include massive hemolysis, blackwater
fever, and multiple organ involvement, including the central nervous system (cerebral
malaria).

Parasitology 415
Parasitology 416
6. The causative agent of cysticercosis is:

A. Taenia solium
B. Taenia saginata
C. Ascaris lumbricoides
D. Trichuris trichiura

ANSWER: A. Taenia solium

Humans may become infected with Taenia solium by either ingesting the larval form or
ova. If ova are ingested the parasite cannot complete the life cycle, and cysticerci
encyst in various tissues including the brain.

Parasitology 417
Parasitology 418
7. The examination of human feces is no help in the detection of:

A. Strongyloides stercoralis
B. Entamoeba histolytica
C. Echinococcus granulosus
D. Ancylostoma duodenale

ANSWER: C. Echinococcus granulosus

The diagnostic stages of Strongyloides, Entamoeba and Ancylostoma can be detected


in the stool of infected patients. The diagnostic stage of Echinococcus granulosus is
not detected in an infected patient’s stool.

Parasitology 419
420
8. A fibrous skin nodule is removed from the back of a patient from
Central America. A microfilaria seen upon microscopic exam of the
nodule is:

A. Wuchereria bancrofti
B. Brugia malayi
C. Onchocerca volvulus
D. Loa loa

ANSWER: C. Onchocerca volvulus

Oncherca volvulus is the only microfilaria that is detected in the skin snips of patients
with raised skin nodules. The microfilaria of Wuchereria, Brugia, and Loa loa are found
in the blood of infected patients.

Parasitology 421
9. Based on the picture below, identify the type of Ascaris
lumbricoides ova.

A. Decorticated
B. Fertilized
C. Unfertilized
D. None of the above

ANSWER: C. Unfertilized

Parasitology 422
10. Charcot–Leyden crystals in stool may be associated with an
immune response and are thought to be formed from the
breakdown products of:

A. Neutrophils
B. Eosinophils
C. Monocytes
D. Lymphocyte

ANSWER: B. Eosinophils

When eosinophils disintegrate, the granules reform into Charcot–Leyden


crystals.

Parasitology 423
11. The examination of sputum may be necessary to diagnose infection with:

A. Paragonimus westermani
B. Trichinella spiralis
C. Wuchereria bancrofti
D. Fasciola hepatica

ANSWER: A. Paragonimus westermani

P. westermani adult worms are found in the lung, and eggs may be coughed up in the sputum.
Consequently, both sputum and stool (if the sputum containing the eggs is swallowed) are the
recommended specimens for examination for the eggs.

Parasitology 424
12. The advantage of thick blood smears for malarial parasites is
to:

A. Improve staining of the organisms


B. Improve detection of the organisms
C. Remove RBC artifacts
D. Remove platelets

ANSWER: B. Improve detection of the organisms

The increased amount of blood placed on the slide of a thick smear for blood parasites
improves sensitivity of the smear. Thick smears should be performed on all requests
for blood parasites.

Parasitology 425
13. Oocysts of Cryptosporidium spp. can be detected in stool specimens
using:

A. Modified Ziehl–Neelsen acid-fast stain


B. Gram stain
C. Methenamine silver stain
D. Trichrome stain

ANSWER: A. Modified Ziehl–Neelsen acid-fast stain

The oocysts of Cryptosporidium spp. can be found and identified using microscopic
examination of fecal smears stained with modified acid-fast stains. They appear as
purple-red-pink round objects, measuring approximately 4–6 μ. Often, the four
sporozoites and residual body can be seen within the oocyst wall.

Parasitology 426
14. Humans can serve as both the intermediate and definitive host in
infections caused by:

A. Enterobius vermicularis
B. Hymenolepis nana
C. Schistosoma japonicum
D. Ascaris lumbricoides

ANSWER: B. Hymenolepis nana

In infections with H. nana, humans serve as both intermediate and definitive hosts.
When ingested, the oncosphere penetrates the intestinal mucosa, develops into the
mature cysticercoid (human is intermediate host), and returns to the gut, where the
adult tapeworm matures (human is definitive host)

Parasitology 427
15. Which parasite causes eosinophilic meningoencephalitis, a form of
larva migrans causing fever, headache, stiff neck, and increased cells in
the spinal fluid?

A. Necator americanus
B. Angiostrongylus cantonensis
C. Ancylostoma braziliense
D. Strongyloides stercoralis

ANSWER: B. Angiostrongylus cantonensis

Eosinophilic meningoencephalitis is a form of larva migrans and is caused by A.


cantonensis, the rat lungworm. This Pacific area infection is associated with CSF
symptoms and sometimes eye involvement

Parasitology 428
Parasitology 429
430
X. General / Laboratory
Operations

431
1. An index of precision is statistically known as:

A. Median
B. Mean
C. Standard Deviation
D. Coefficient of variation

ANSWER: D. Coefficient of variation

Precision is the closeness of agreement among replicate measurements, or


reproducibility. The coefficient of variation, a more useful measure of reproducibility, is
the measure of relative random error expressed as percentage.

General / Lab operations 432


2. Diagnostic specificity is defined as the percentage of individuals:

A. With a given disease who have a positive result by a given test


B. Without a given disease who have a negative results by a given test
C. With a given disease who have a negative result by a given test
D. Without a given disease who have a positive result by a given test

ANSWER: B. Without a given disease who have a negative results by a given test

Specificity is defined as negativity in the absence of disease.

Sensitivity refers to a test's ability to designate an individual with disease as positive. A


highly sensitive test means that there are few false negative results, and thus fewer
cases of disease are missed.

General / Lab operations 433


General / Lab operations 434
3. Which statement about Proficiency Testing is true?

A. Results can be compared to another hospital prior to submission if that hospital is


in your system
B. Results between 2 technologists can be averaged
C. CAP requires duplicate testing to ensure good instrument performance
D. It is necessary to assess results even if a PT challenge is ungraded

ANSWER: D. It is necessary to assess results even if a PT challenge is ungraded

CAP general checklist 1126: “Does the laboratory have a procedure for assessing its
performance on PT challenges that were not graded…”

General / Lab operations 435


FYI !
Proficiency Testing
Determines the performance of individual laboratories for
specific tests or measurements and is used to monitor
laboratories’ continuing performance
◦ Verify laboratory’s accuracy and reliability
◦ CLIA mandated
◦ By CMS approved PT program
◦ Testing for each CLIA license
◦ Sample tested as patient samples
◦ Discussion or re-routing of samples to other labs
Records – minimum of two years
4. CAP requires refrigerator temperatures to be recorded:

A. Daily
B. Weekly
C. Monthly
D. Periodically

ANSWER: A. Daily

Gen 51045: Are refrigerator/freezer temperatures checked and recorded daily?

General / Lab operations 437


5. After receiving appropriate training, the first step in using a fire
extinguisher is to:

A. Sweep the flow of the hose from side to side


B. Pull the pin
C. Squeeze the top handle or lever
D. Aim the hose at the base of the fire

ANSWER: B. Pull the pin


REMEMBER!
P Pull the pin
A Aim the hose at the base of the fire
S Squeeze the top handle or lever
S Sweep the flow of the hose from side to side

General / Lab operations 438


6. Class C fires involve:

A. Grease and Oil


B. Xylene and Alcohol
C. Paper, wood and plastics
D. Electrical equipment

ANSWER: D. Electrical Equipment

Grease and Oil: Class B


Xylene and Alcohol: Class B
Paper, wood and plastics: Class A
(Forbes 2007)

General / Lab operations 439


7. Which of the following is the best choice for decontaminating bench
tops contaminated by the AIDS virus?

A. Sodium hypochlorite bleach


B. Formalin
C. A quaternary ammonium compound
D. 100% alcohol

ANSWER: A. Sodium hypochlorite bleach

A 10% bleach is an effective and economical disinfectant, which inactivates HBV in 10


minutes and HIV in 2 minutes. (Henry 2006)

General / Lab operations 440


8. Infection rate is highest for laboratory professionals exposed to blood
and body fluids containing?

A. Hepatitis A
B. Hepatitis B
C. CMV
D. HIV

ANSWER: B. Hepatitis B

Hepatitis B infection is a global public health problem and is one of the most common
infectious diseases in the world. (Murray 2007)

General / Lab operations 441


9. Filters generally used in biological safety cabinets to protect the
laboratory worker from particulates and aerosols generated by
microbiology manipulations are:

A. Fiberglass
B. HEPA
C. APTA
D. Charcoal

ANSWER: B. HEPA

Microbiological hazards are contained using a biological safety cabinet with air
exhausting through a HEPA filter. (Henry 2006)

General / Lab operations 442


10. This symbol represents:

A. Biohazard
B. Radiation Hazard
C. Chemical Hazard
D. Environmental Hazard

ANSWER: A. Biohazard (Fleming 2006)

General / Lab operations 443


12. The volume of 25% stock sulfosalicylic acid needed to prepare
100 mL of 5% working solution is:

A. 1.25 mL
B. 5 mL
C. 20 mL
D. 50 mL

ANSWER: C. 20 mL

The most commonly used equation for preparing suspension solutions is:
V1 x C1 = V2 x C2
(100) X (5) = (x) x (25)
(Kaplan 2003)

General / Lab operations 444


13. Four mL of water are added to 1 mL of serum. This represent
which of the following serum dilutions?

A. 1:3
B. 1:4
C. 1:5
D. 1:6

ANSWER: C. 1:5

Simple dilutions are ratios of 2 volumes, which involve a single substance diluted with
one other substance. In this case, 1mL solution A is added to 4mL solution B (ratio 1:4)
for a total volume of 5mL. This represents a dilution of 1:5. (Campbell 1997)

General / Lab operations 445


14. A technologist spilled 10 gallons of formaldehyde on the floor.
After determining the chemical poses a significant health hazard,
the first action step would be to:

A. Notify emergency assistance


B. Control the spill with appropriate absorbent material
C. Evacuate the area
D. Wear appropriate personal protective equipment

ANSWER: C. Evacuate the area

The other options from the choices are secondary responses after evacuating the area.
(BOC 5th Edition)

General / Lab operations 446


15. Which of the following questions can legally asked on an
employment application:

A. Are you a US citizen?


B. What is your date of birth?
C. Is your wife/husband employed full time?
D. Do you have any dependents?

ANSWER: A. Are you a US citizen?

Questions that could be discriminatory in nature (e.g., nationality, marital status,


dependents, religion, affiliations, sexual orientation, or physical or mental disabilities)
should be avoided on the employment application or during the interview process,
unless the question is relevant or pertinent to the particular job; otherwise, it can be
considered illegal. (Darrah 2007)

General / Lab operations 447


TIPS
Memorize/Familiarize the formulas

General / Lab operations 448


TIPS
Be familiar with the laboratory signs/symbols

General / Lab operations 449


XI. Road to USA Tips

450
REQUIREMENTS for US applications:

1. ASCPi (see signing up guide for ASCPi by Sir Norman)

2. IELTS
British council: https://ielts.britishcouncil.org/Default.aspx
or IDP: https://my.ieltsessentials.com/IELTS

3. Visa screen
CGFNS: https://start.cgfns.org/applicants/

4. World Education Services (Course by course analysis) for states that requires LICENSE only
Choose PROFESSIONAL LICENSE or CERTIFICATION evaluation

WES: https://www.wes.org/evaluations-and-fees/

**Currently, California, Florida, Georgia, Hawaii, Louisiana, Montana, Nevada, New York, North Dakota,
Puerto Rico, Tennessee, and West Virginia require licensure.

451
Once you are done it’s time for JOB HUNTING:
Currently, we are sponsored through H1B working visa.

2 categories of sponsoring facilities:

For profit:
❖ H1B visa under CAP, all applicants will be subjected to lottery every April 1st

Non-profit:
❖ CAP-exempt H1B visa can be applied anytime of the year, once approved you are good to
work
❖ If coming from Philippines: there are separate requirements from POEA (OEC, PDOS and
Medical)

452
Afraid that your ASCPi will expire?
Don’t be!
You can get free CE units from the following:
1. PAMET-USA, Inc. and it’s chapters offer free CE units through live webinars. Keep
an eye for those!

2. Register on the mayo clinic website for FREE and get your CE units for free
✓ https://www.mayocliniclabs.com/register/index.php
After registering click on that link below, choose the ones with CREDIT OFFERED.
https://news.mayocliniclabs.com/category/mml-education/on-demand-programs/virtual-visiting-faculty/

3. There are FREE CE units offered at ASCP.org


✓ Under Education tab, go to find education then sort by price. Add to cart all
the 0.00 priced courses then you can launch it later.
And many more! In fact, once you start working here your facility will pay for it usually.
453
ASCP.ORG

454
Check these links for job opportunities:

https://www.pametusainc.org/job-opportunities

https://labmindstaffing.com/

455
REFERENCES/SOURCES:

❖ Harmening, D. (2012). Modern blood banking & transfusion practices. Philadelphia: F.A. Davis.
❖ Mayr, W. R. (2003). AABB Technical Manual. Vox Sanguinis,84(4), 339-339. doi:10.1046/j.1423-0410.2003.00293.x
❖ ASCP Board of Certification. Revised June 2020
❖ CHS interns’ complied recalls 2019-2021
❖ Clinical laboratory Science Review: A Bottom Line Approach 5th Edition by Patsy Jarreau, MHS, MLS (ASCP), et. al.
❖ Rodak’s Hematology 6th edition
❖ https://www.cellavision.com/en/cellavision-cellatlas/other-findings
❖ BOC Study guide 5th edition by Patricia Tanabe, MPA, MLS (ASCP)CM and E. Blair Holladay, PhD, SCT (ASCP)CM
❖ Mayr, W. R. (2003). AABB Technical Manual. Vox Sanguinis,84(4), 339-339. doi:10.1046/j.1423-0410.2003.00293.
❖ Immuno Concepts. Online Training.
❖ Turgeon Mary Louise. Immunology and Serology in Laboratory Medicine. 6th edition.

456
Credits to the owner

457
THANK YOU and GOOD LUCK!

458

You might also like